Download as pdf or txt
Download as pdf or txt
You are on page 1of 107

ILP FRESHERS- 2020 TEST

Exam Title :
41 ...
Email
Contact

QUESTION 1. MjI2MTgwK1BBUlRIK3BhcnRodW1yYWxpeWE1NUBnbWFpbC5jb20rODE0MDA3MTI2OFFVRV
NUSU9OIDA=
The Bharata Itihasa Samshodhaka Mandala (BISM) was founded in 1910 by:

a) Vishwanath Kashinath Rajwade


b) Ramakrishna Gopal Bhandarkar
c) Rajendra Lal Mitra
d) K.A. Nilakanta Sastri
Correct Answer: A
Your Answer: Unanswered
Explanation

Solution (a)

Basic Information:

· The Bharata Itihasa Samshodhaka Mandala (BISM) owed its origins to the stalwart
Vishwanath Kashinath Rajwade.

· He founded this institution in 1910 in Pune with the support of K C Mehendale.

· Vishwanath Kashinath Rajwade (24 June 1863 – 31 December 1926), popularly known as
Itihasacharya Rajwade, was a historian, scholar, writer, commentator and orator from
Maharashtra.

· In 1895 he started a Marathi magazine called Bhashantar (meaning ‘translation’) through


which he brought works of Western historians and scholars such as Plato, Aristotle, and Edward
Gibbon, and also Indian scholars like Shankaracharya in Marathi.

· After his sudden death in 1926, ‘Rajwade Sanshodhak Mandal’ was founded at Dhule

· His noted work: Marathyanchya Itihasachi Sadhane (History of Marathas), Radha Madhav
Vilas Champu (Biography of Shahaji), Bharatiya Vivah Sansthecha Itihas (History of Indian
matrimony), Aitihasik Prastavana (Historical Prefaces)

· The BISM organised an All India Congress in 1935 to celebrate its silver jubilee. This event
resulted in the inception of the Indian History Congress (IHC).

· The Bhandarkar Oriental Research Institute founded in 1917 by R G Bhandarkar.

QUESTION 2. MjI2MTgwK1BBUlRIK3BhcnRodW1yYWxpeWE1NUBnbWFpbC5jb20rODE0MDA3MTI2OFFVRV
NUSU9OIDE=
The Battle of Bhima-Koregaon was part of which of the following war?

a) Fourth Anglo-Mysore war


b) Second Anglo-Maratha war
c) Third Anglo-Maratha war
d) None of the Above

IASbaba
Score:
Web: http://ilp.iasbaba.com/
41.00 /
Email: ilp@iasbaba.com
Page 1 200
ILP FRESHERS- 2020 TEST
Exam Title :
41 ...
Email
Contact

Correct Answer: C
Your Answer: D
Explanation

Solution (c)

Basic Information:

· 202nd anniversary of the Bhima-Koregaon battle of 1818 was observed on January 1, 2020.

· The last battle of the Third Anglo-Maratha war was fought at Koregaon on the banks of Bhima
river, hence the name Bhima Koregaon battle, on January 1, 1818

· The three wars between the British East India Company and the Maratha Empire are referred
to as the great Maratha Wars or the Anglo-Maratha Wars.

· The Third Anglo-Maratha War (1817-1818) because of conflict between the British and the
Pindaris. The British suspected that the Marathas were providing help to the Pindaris, who
were mercenaries fighting for the Marathas.

· Peshwa intended to attack Pune, but British dispatched troops from Shirur, resulting in the
Battle of Bhima-Koregaon on January 1, 1818.

· The battle was decisively won by Company forces. The dead Company soldiers of Indian origin
included 22 Mahars, 16 Marathas, 8 Rajputs, 2 Muslims, and 2 Jews.

· On 1 January 1927, B.R. Ambedkar visited the memorial obelisk erected on the spot of Bima-
Koregaon, which bears the names of the dead including nearly two dozen Mahar soldiers.

( Source: https://www.thehindu.com/thread/politics-and-policy/what-the-battle-of-bhima-
koregaon-meant/article22378086.ece )

QUESTION 3. MjI2MTgwK1BBUlRIK3BhcnRodW1yYWxpeWE1NUBnbWFpbC5jb20rODE0MDA3MTI2OFFVRV
NUSU9OIDI=
Consider the following statements:

1. She was first Indian woman to become a teacher.

2. She also worked towards preventing female infanticide and set up a home, Balhatya
Pratibandhak Griha.

3. She wrote Bavan Kashi Subodh Ratnakar.

The above mentioned statements are associated with which of the following social reformer:

a) Fatima Sheikh
b) Pandita Ramabai
c) Ramabai Ranade
d) Savitribai Phule
Correct Answer: D

IASbaba
Score:
Web: http://ilp.iasbaba.com/
41.00 /
Email: ilp@iasbaba.com
Page 2 200
ILP FRESHERS- 2020 TEST
Exam Title :
41 ...
Email
Contact

Your Answer: Unanswered


Explanation

Solution (d)

Basic Information:

· Savitribai Phule was born on 3 January 1831 at Naigaon in Maharashtra’s Satara district. She
was the eldest daughter of Lakshmi and Khandoji Neveshe Patil, both of whom belonged to the
Mali community, now an Other Backward Caste (OBC). At the age of 9, she was married to 13-
year-old Jyotirao Phule.

· Jyotiba, at the age of 21, and Savitri, 17, opened a school for women in 1848. It was the
country’s first school for women started by Indians.

· Savitribai Phule was taught by Jyotiba at their home. Later, she took a teacher’s training
course at an institute run by an American missionary in Ahmednagar and in Pune’s Normal
School.

· Fatima Sheikh went along with Savitribai to the Normal School and they both graduated
together. Fatima and Savitribai opened a school in Usman Sheikh’s house in Pune in 1849.

· In the 1850s, the Phule couple initiated two educational trusts—the Native Female School,
Pune and The Society for Promoting the Education of Mahars, Mangs and Etceteras—which
came to have many schools under them.

· Savitribai was also a fiery author and poetess. She published Kavya Phule in 1854 and Bavan
Kashi Subodh Ratnakar in 1892.

· In 1852, Savitribai, a staunch feminist, started the Mahila Seva Mandal to raise awareness
about women’s rights.

· She also started the Home for the Prevention of Infanticide in her house, a place where
Brahmin widows could deliver their babies safely.

· She initiated the first Satyashodhak marriage—a marriage without dowry, Brahmin priests or
Brahminical rituals.

· Savitribai broke yet another taboo when she led the funeral procession of her husband.

(Source: https://www.livemint.com/Leisure/DmR1fQSnVD62p4D3eyq9mO/The-life-and-times-of-
Savitribai-Phule.html )

QUESTION 4. MjI2MTgwK1BBUlRIK3BhcnRodW1yYWxpeWE1NUBnbWFpbC5jb20rODE0MDA3MTI2OFFVRV
USU9OIDM=
Consider the following statements:

1. He reduced the number of elected representatives of the Calcutta Corporation through the
Calcutta Municipal Amendment Act of 1899.

2. Through the Indian Universities Act of 1904 he placed Universities in India under
governmental control.

IASbaba
Score:
Web: http://ilp.iasbaba.com/
41.00 /
Email: ilp@iasbaba.com
Page 3 200
ILP FRESHERS- 2020 TEST
Exam Title :
41 ...
Email
Contact

3. Under the Indian Official Secrets Amendment Act of 1904 he restricted freedom of press.

Which of the following Governor General is associated with the above statements?

a) Lord Dalhousie
b) Lord Curzon
c) Lord Lytton
d) Lord Monto
Correct Answer: B
Your Answer: Unanswered
Explanation

Solution (b)

Basic Information:

Lord Curzon (1899-1905), initiated a number of unpopular legislative and administrative


measures, which hurt the susceptibilities of the educated Indians.

· The reconstitution of the Calcutta Corporation through the Calcutta Municipal Amendment Act
of 1899 reduced the number of elected representatives in it;

· the Indian Universities Act of 1904 placed Calcutta University under the most complete
governmental control;

· the Indian Official Secrets Amendment Act of 1904 further restricted press freedom.

· The last in the series was the partition of Bengal in 1905, designed to weaken the Bengali
nationalists who allegedly controlled the Congress.

Some of his popular initiatives:

· He established The Agriculture Research Institute in Pusa (Bihar – Bengal Presidency)

· He instituted a Police Commission in 1902 under the chairmanship of Sir Andrew Frazer.

· He passed a law called the Ancient Monuments Act, 1904

(Source: “From Plassey to Partition: A History of Modern India”)

QUESTION 5. MjI2MTgwK1BBUlRIK3BhcnRodW1yYWxpeWE1NUBnbWFpbC5jb20rODE0MDA3MTI2OFFVRV
NUSU9OIDQ=
Consider the following the statements with reference to Madan Mohan Malaviya:

1. He contributed in establishing the Benares Hindu University.

2. He did not participate in the Civil Disobedience Movement

3. He was posthumously conferred with Bharat Ratna, the country's highest civilian award.

Which of the statements given above is/are correct?

IASbaba
Score:
Web: http://ilp.iasbaba.com/
41.00 /
Email: ilp@iasbaba.com
Page 4 200
ILP FRESHERS- 2020 TEST
Exam Title :
41 ...
Email
Contact

a) 1 and 2 Only
b) 2 and 3 Only
c) 1 and 3 Only
d) 1, 2 and 3
Correct Answer: C
Your Answer: D
Explanation

Solution (c)

Basic Information:

· December 25th is the birth anniversary of Madan Mohan Malaviya, the famed Indian
educationist and freedom fighter who is also called ‘Mahamana’.

· He was one of the torchbearers of the freedom struggle — acting as a bridge between the
Moderates and the Extremists in the Congress.

· Drawn to politics, Malaviya joined the Indian National Congress at its Calcutta session in 1886
— it had been founded a year previously at the Gokuldas Tejpal Sanskrit College in Mumbai.

· Malaviya rose up the ranks, and became president four times — in 1909 (Lahore), in 1918
(Delhi), in 1930 (Delhi), and in 1932 (Calcutta). Malaviya was part of the Congress for almost 50
years.

· Malaviya was one of the early leaders of the Hindu Mahasabha, and helped found it in 1906.

Statement Analysis:

Statement 1 Statement 2 Statement 3

Correct Incorrect Correct

He along with Anne Besant In 1930, when Mahatma


In 2015, the government
and others established Gandhi launched the Salt
bestowed Malaviya with the
Banaras Hindu University Satyagraha and the Civil
Bharat Ratna, India’s
1916, he served as Vice- Disobedience Movement,
highest civilian honour, 68
Chancellor from 1919 to he participated in it and
years after his death.
1938. courted arrest.

(Source: https://indianexpress.com/article/explained/who-was-pandit-madan-mohan-malaviya-
freedom-fighter-and-educationist-6184723/ )

QUESTION 6. MjI2MTgwK1BBUlRIK3BhcnRodW1yYWxpeWE1NUBnbWFpbC5jb20rODE0MDA3MTI2OFFVRV
NUSU9OIDU=
Consider the following statements:

IASbaba
Score:
Web: http://ilp.iasbaba.com/
41.00 /
Email: ilp@iasbaba.com
Page 5 200
ILP FRESHERS- 2020 TEST
Exam Title :
41 ...
Email
Contact

1. The Indian History Congress (IHC) was born with about 50 delegates in Pune, Maharashtra.

2. The first All India Oriental Conference in November 1919 was held in Bengal.

Which of the statements given above is/are incorrect?

a) 1 Only
b) 2 Only
c) Both 1 and 2
d) Neither 1 nor 2
Correct Answer: B
Your Answer: Unanswered
Explanation

Solution (b)

Basic Information:

· Recently, the 80th session of the Indian History Congress (IHC) was held at Kannur, Kerala,
between 28th and 30th December 2019.

· IHC was born with about 50 delegates to be an academy that regulated the standard of works
of history produced in India and would promote impartial and substantive history.

· Historians such as Datto Vaman Potdar, Surendra Nath Sen (who later became the first
director of National Archives of India), and Sir Shafaat Ahmad Khan attended the first session

· In fact, in 1946, it petitioned the government to allow researchers access to archives. Later, in
1977, the IHC cautioned against the use of communal rhetoric in public life and the dangers of
political appropriation of monuments.

Statement Analysis:

Statement 1 Statement 2

Correct Incorrect

The BISM organised an All India Congress


The first All India Oriental Conference in
in 1935 to celebrate its silver jubilee. The
November 1919 in Pune under the
event resulted in the inception of the IHC.
Bhandarkar Institute’s auspices focused
The event organised in the assembly hall
exclusively on ancient Indian history.
of Parasurambhau College, Pune

(Source: https://indianexpress.com/article/opinion/columns/indian-history-congress-that-
wasnt-5514300/ )

IASbaba
Score:
Web: http://ilp.iasbaba.com/
41.00 /
Email: ilp@iasbaba.com
Page 6 200
ILP FRESHERS- 2020 TEST
Exam Title :
41 ...
Email
Contact

QUESTION 7. MjI2MTgwK1BBUlRIK3BhcnRodW1yYWxpeWE1NUBnbWFpbC5jb20rODE0MDA3MTI2OFFVRV
NUSU9OIDY=
Consider the following statements with reference to the Arya Samaj:

1. The Arya Samaj developed the concept of suddhi, which aimed at reconversion from
Christianity, Islam and Sikhism.

2. It offered a sharp criticism of the existing Hindu practices, like idolatry, polytheism, child
marriage, widow celibacy, dominance of Brahmans and caste system.

3. Lala Lajpat Rai became follower of Vivekanada, the founder of the Arya Samaj

Which of the statements given above is/are correct?

a) 1 and 2 Only
b) 2 and 3 Only
c) 1 and 3 Only
d) 1, 2 and 3
Correct Answer: A
Your Answer: A
Explanation

Solution (a)

Basic Information:

· The Arya Samaj Movement, revivalist in form though not in content, was the result of a
reaction to western influences. Its founder, Dayanand.Saraswati (or Mulshankar, 1824-83) was
born in the old Morvi state in Gujarat.

· Dayanand's views were published in his famous work, Satyarth Prakash (The True Exposition).

· He took inspiration from the Vedas and considered them to be "India's Rock of Ages", the
infallible and the true original seed of Hinduism

· He gave the slogan "Back to the Vedas".

· Arya Samaj offered a sharp criticism of the existing Hindu practices, like idolatry, polytheism,
child marriage, widow celibacy, foreign travel, dominance of Brahmans and caste system.

· Lala Lajpat Rai was associated with Arya Samaj and was editor of Arya Gazette, which he set
up during his student time.

Statement Analysis:

Statement 1 Statement 2 Statement 3

Correct Correct Incorrect

IASbaba
Score:
Web: http://ilp.iasbaba.com/
41.00 /
Email: ilp@iasbaba.com
Page 7 200
ILP FRESHERS- 2020 TEST
Exam Title :
41 ...
Email
Contact

He denounced meaningless
rites, worship of the images
The Shuddhi programme of different gods and
started by Arya Samaj, goddesses which split the
Arya Samaj was established
aimed to open the doors of people into numerous
in 1875 by Dayanand
Hinduism for those who belligerent sects,
Saraswati.
had embraced other repudiated the authority of
religions. the Brahmins and launched
a crusade against all
religious superstitions.

(Source: Spectrum)

QUESTION 8. MjI2MTgwK1BBUlRIK3BhcnRodW1yYWxpeWE1NUBnbWFpbC5jb20rODE0MDA3MTI2OFFVRV
NUSU9OIDc=
“Poverty and UnBritish Rule in India” was written by

a) Dadabhai Naoroji
b) M G Ranade
c) R C Dutt
d) Mahatma Gandhi
Correct Answer: A
Your Answer: A
Explanation

Solution (a)

Basic Information:

· September 4, 2019 was the 194th birth anniversary of Dadabhai Naoroji, the “Grand Old Man
of India”, who was among the first leaders who stirred national consciousness in the country.

· Born in 1825 at Navsari, in present-day Gujarat, Naoroji.

· He was closely involved with the Indian National Congress in its early phase, and served as
the first Indian member of the British parliament.

· In 1895, he became a member of the royal commission on Indian expenditure.

· Naoroji’s lasting intellectual contribution was to expound the ‘Drain Theory’ disseminating it
in his 1901 book ‘Poverty and Un-British Rule in India’.

· Naoroji argued that imperial Britain was draining away India’s wealth to itself through
exploitative economic policies, including India’s rule by foreigners; the heavy financial burden
of the British civil and military apparatus in India; the exploitation of the country due to free
trade; non-Indians taking away the money that they earned in India; and the interest that India
paid on its public debt held in Britain.

IASbaba
Score:
Web: http://ilp.iasbaba.com/
41.00 /
Email: ilp@iasbaba.com
Page 8 200
ILP FRESHERS- 2020 TEST
Exam Title :
41 ...
Email
Contact

(Source: https://indianexpress.com/article/explained/explained-remembering-dadabhai-naoroji-
on-his-birthday-5966633/ )

QUESTION 9. MjI2MTgwK1BBUlRIK3BhcnRodW1yYWxpeWE1NUBnbWFpbC5jb20rODE0MDA3MTI2OFFVRV
NUSU9OIDg=
Consider the following statements with reference to the Third Battel of Panipat (1761):

1. It was fought between forces of Maratha Empire and Durrani Empire (Afghanistan)

2. Bajirao Peshwa was the Commander in Chief of Maratha army.

3. Shah Alam II was the Mughal emperor during the Battle.

Which of the statements given above is/are correct?

a) 1 Only
b) 2 and 3 Only
c) 1 and 3 Only
d) 1, 2 and 3
Correct Answer: C
Your Answer: C
Explanation

Solution (c)

Basic Information:

· Abdali was supported by two Indian allies—the Rohillas Najib-ud-daulah, Afghans of the Doab
region and Shuja-ud-Daula-the Nawab of Awadh.

· The Marathas were defeated in the battle, with 40-50,000 of their troops killed, while Abdali’s
army is estimated to have suffered around 20,000 casualties.

· It had great impact on Maratha empire prospects. Many important Maratha chiefs including
Bhau, son of the Peshwa, Viswas Rao, Jaswant Rao Pawar, Tukoji Sindhia etc. were killed in the
battle.

· The Maratha rise was checked but they retook Delhi ten years later under Peshwa Madhavrao.

· The First Battle of Panipat, in 1526 was fought between Babur and Ibrahim lodhi. laid the
foundation of the Mughal Empire in India.

· The Second Battle of Panipat, in 1556 was fought between Akbar and King Hemu
‘Vikramaditya’.

Statement Analysis:

IASbaba
Score:
Web: http://ilp.iasbaba.com/
41.00 /
Email: ilp@iasbaba.com
Page 9 200
ILP FRESHERS- 2020 TEST
Exam Title :
41 ...
Email
Contact

Statement 1 Statement 2 Statement 3

Correct Incorrect Correct

Sadashivrao Bhau was the


The third battle of Panipat
Commander in Chief of
which was fought between
Maratha army and Balji Shah Alam II was Mughal
the Marathas and Ahmad
Barjirao the successor of emperor and was under
Shah Abdali, the ruler of
Bajirao Peshwa was the protection of Marathas.
Durrani Empire of
Peshwa at the time of
Afghanistan.
Battle.

(Source: https://theprint.in/opinion/panipat-was-a-bloody-military-debacle-for-marathas-will-
patriotism-high-india-see-the-film/317220/ )

QUESTION 10. MjI2MTgwK1BBUlRIK3BhcnRodW1yYWxpeWE1NUBnbWFpbC5jb20rODE0MDA3MTI2OFFVR


NUSU9OIDk=
Consider the following statements:

1. The queen of Jhansi died while fighting a squadron of the 8th Hussars under Captain
Heneage

2. Jhansi was the first princely state to be annexed by British under Doctrine of Lapse.

Which of the statements given above is/are correct?

a) 1 Only
b) 2 Only
c) Both 1 and 2
d) Neither 1 nor 2
Correct Answer: A
Your Answer: D
Explanation

Solution (a)

Basic Information:

· Rani Lakshmibai born on November 19, 1828, as Manikarnika Tambe in Varanasi. Rani was
married to the King of Jhansi, Raja Gangadhar Newalkar in 1842.

· She gave birth to a boy, later named Damodar Rao, in 1851, who died after four months. The
Maharaja adopted a child called Anand Rao, the son of Gangadhar Rao's cousin, who was
renamed Damodar Rao, on the day before the Maharaja died.

IASbaba
Score:
Web: http://ilp.iasbaba.com/
41.00 /
Email: ilp@iasbaba.com
Page 10 200
ILP FRESHERS- 2020 TEST
Exam Title :
41 ...
Email
Contact

· Under Governor-General Lord Dalhousie, the Doctrine of Lapse was applied, rejecting
Damodar Rao's claim to the throne and annexing the state to its territories.

· In March 1854, Rani Lakshmibai was given an annual pension of Rs. 60,000 and ordered to
leave the palace and the fort.

· The Doctrine of Lapse was an annexation policy followed widely by Lord Dalhousie when he
was India's Governor-General from 1848 to 1856.

· Under it, any princely state under the direct or indirect (as a vassal) control of the East India
Company where the ruler did not have a legal male heir would be annexed by the company.

· Also any adopted son of the Indian ruler could not be proclaimed as heir to the kingdom.

Statement Analysis:

Statement 1 Statement 2

Correct Incorrect

On 17 June in Kotah-ki-Serai near the Satara (1848) was the first princely state
Phool Bagh of Gwalior, a squadron of the to be annexed under Doctrine of Lapse
8th (King's Royal Irish) Hussars, under followed by Jaitpur and Sambalpur
Captain Heneage, fought the large Indian (1849), Baghat (1850), Chota Udaipur
force commanded by Rani Lakshmibai. (1852), Jhansi (1853), and Nagpur (1854).

(Spectrum)

QUESTION 11. MjI2MTgwK1BBUlRIK3BhcnRodW1yYWxpeWE1NUBnbWFpbC5jb20rODE0MDA3MTI2OFFVR


NUSU9OIDEw
Which of the following Indian revolutionary assassinated Colonel Reginald Edward Harry Dyer?

a) Madan Lal Dhingra


b) Udham Singh
c) Khudiram Bose
d) None of the above
Correct Answer: D
Your Answer: Unanswered
Explanation

Solution (d)

Basic Information:

· Dyer died of cerebral haemorrhage and arteriosclerosis in 1927.

IASbaba
Score:
Web: http://ilp.iasbaba.com/
41.00 /
Email: ilp@iasbaba.com
Page 11 200
ILP FRESHERS- 2020 TEST
Exam Title :
41 ...
Email
Contact

· On March 13, 1940 Udham Singh shot O’Dwyer at a meeting of the East India Association and
the Royal Central Asian Society at Caxton Hill. He was a political activist who got associated
with the Ghadar Party while in the US.

· He considered O’Dwyer to be the actual perpetrator as he the Lieutenant Governor of Punjab


in 1919.

· On April 13, 1919, Baisakhi day, Brigadier General (temporary rank) Reginald Dyer without
warning ordered troops to fire at crowd gathered at Jallianwala Bagh to protest against arrest
of leaders like Saiffudin Kitchlew under Rowlatt Act.

· The Anarchical and Revolutionary Crimes Act of 1919, better known as the Rowlatt Act, came
into force a month before the massacre in Jallianwala Bagh.

· A Disorders Inquiry Committee, under Chairmanship of Lord Hunter, was set up. The
committee selectively criticised Dyer but let off the Lieutenant Governor, Michael O’Dwyer.

(Source: Spectrum)

QUESTION 12. MjI2MTgwK1BBUlRIK3BhcnRodW1yYWxpeWE1NUBnbWFpbC5jb20rODE0MDA3MTI2OFFVR


NUSU9OIDEx
Consider the following statements:

1. Azad Hind Fauj had two avatars – under Captain Mohan Singh and Subhash Chandra Bose.

2. Capitan Mohan Singh became the first commander of the Indian National Army.

3. Bose started Azad Hind Radio as part of Germany’s radio service, which first aired on 7
January 1942 in Singapore.

Which of the statements given above is/are correct?

a) 1 and 2 Only
b) 2 Only
c) 1 and 3 Only
d) 2 and 3 Only
Correct Answer: A
Your Answer: A
Explanation

Solution (a)

Basic Information:

· Japan sent Major Fujiwara to Southeast Asia to contact expatriate Indians who were
organising themselves into the Indian Independence Leagues under the leadership of men like
Pritam Singh.

· By September 1942, the INA was formally in existence under Mohan Singh.

IASbaba
Score:
Web: http://ilp.iasbaba.com/
41.00 /
Email: ilp@iasbaba.com
Page 12 200
ILP FRESHERS- 2020 TEST
Exam Title :
41 ...
Email
Contact

· As Mohan Singh insisted on autonomy and allied status, he was removed from command and
put under arrest. Rash Behari Bose tried to hold the banner for some time, but he was then too
aged for the task.

· After a long and arduous submarine voyage, in May 1943 Bose arrived in Southeast Asia.

· He became the supreme commander of its army, the Azad Hind Fauj (Free India Army) or the
Indian National Army, which recruited around forty thousand men by 1945. and had a women's
regiment named after the legendary Rani of Jhansi of 1857 fame.

· In October 1943, he established a Provisional Government of Free India, which was


immediately recognised by Japan and later by eight other governments, including Germany and
Fascist Italy.

Statement Analysis:

Statement 1 Statement 2 Statement 3

Correct Correct Incorrect

In December 1941, Captain


Mohan Singh, agreed to Captain Mohan Singh, a Subhas Bose's Azad Hind
cooperate with Japan to young officer of the Punjab Radio, broadcast from
raise an Indian army with Regiment of the British Berlin from March 1942.
POWs to march alongside Indian Army who had Bose continued his
the Japanese to liberate surrendered to the broadcast and overseas
India. But, by the beginning Japanese, agreed to transmission to India from
of 1943 the first INA cooperate with Fuziwara to Germany, and eventually
experiment virtually raise an Indian army with Southeast Asia, right up
collapsed. In May 1943 POWs to march alongside until June 1945, even after
Bose arrived in Southeast the Japanese to liberate his Indian National Army
Asia and immediately took India. Thus becoming 1 st C was defeated by the British
control of second avatar of ommander of INA. in 1944.
INA.

QUESTION 13. MjI2MTgwK1BBUlRIK3BhcnRodW1yYWxpeWE1NUBnbWFpbC5jb20rODE0MDA3MTI2OFFVR


NUSU9OIDEy
The “Servants of India Society” was founded by

a) Bal Gangadhar Tilakk


b) M K Gandhi
c) Gopala Krishna Gokhale
d) Dadabhai Naoroji
Correct Answer: C
Your Answer: C

IASbaba
Score:
Web: http://ilp.iasbaba.com/
41.00 /
Email: ilp@iasbaba.com
Page 13 200
ILP FRESHERS- 2020 TEST
Exam Title :
41 ...
Email
Contact

Explanation

Solution (c)

Basic Information:

· In 1905, Gokhale founded the 'Servants of India Society' with the object of training men to
devote themselves to the service of India as national missionaries and to promote by all
constitutional means the national interests of the Indian people.

· In the field of famine relief, union organisation, cooperatives and uplift of tribals and
depressed, the Society did commendable work.

· From 1911, it also published its newsletter titled “Hitavada” in English from Nagpur. The
society still exists and has its headquarters in Pune.

· After Gokhale's death (1915), Srinivasa Shastri took over as president.

· Gopal Krishna Gokhale was born on 9 May 1866 at Katulk in Ratnagiri. Gokhale was greatly
influenced by M. G. Ranade, whom he regarded as his master in political and public life,
Dadabhai Naoroji, who was his hero, and Pherozeshah Mehta.

· Gokhale joined the Deccan Education Society, Poona, as a Life Member. When the Fergusson
College was opened in 1885, he was called upon to lecture there. He retired in 1902 specifically
to devote himself to public life.

(Source: Spectrum)

QUESTION 14. MjI2MTgwK1BBUlRIK3BhcnRodW1yYWxpeWE1NUBnbWFpbC5jb20rODE0MDA3MTI2OFFVR


NUSU9OIDEz
Consider the following statements with reference to formation of Interim Government in 1946:

1. It was the only such cabinet in India’s history in which arch-rivals Congress and the Muslim
League shared power at the Centre.

2. Eventually all of the five ministries reserved for Muslims were occupied by Muslim league
representatives.

3. Initially, Maulana Abul Kalam Azad was not inducted into the cabinet.

Which of the statements given above is/are correct?

a) 1 and 2 Only
b) 2 and 3Only
c) 1 and 3 Only
d) 1, 2 and 3 Only
Correct Answer: D
Your Answer: Unanswered
Explanation

IASbaba
Score:
Web: http://ilp.iasbaba.com/
41.00 /
Email: ilp@iasbaba.com
Page 14 200
ILP FRESHERS- 2020 TEST
Exam Title :
41 ...
Email
Contact

Solution (d)

Basic Information:

· in 1946, the interim government of India led by Jawaharlal Nehru was formed. It was the only
such cabinet in India’s history in which arch-rivals Congress and the Muslim League shared
power at the Centre.

· The interim government functioned with a great degree of autonomy, and remained in power
until the end of British rule, after which it was succeeded by the Dominions of India and
Pakistan.

· The Cripps mission in 1942, made a number of attempts to form an interim government in
India.

· In 1946, elections to the Constituent Assembly were held following the proposals of the British
Cabinet Mission dispatched by the British Prime Minister Clement Attlee.

· Viceroy Wavell subsequently called upon Indian representatives to join the interim
government.

· The Interim government functioned according to the older Government of India Act of 1919.

Statement Analysis:

Statement 1 Statement 2 Statement 3

Correct Correct Incorrect

Intially as League decided


to sit out, three of the five
ministries reserved for
Muslims were occupied by
Asaf Ali, Sir Shafaat Ahmad
The Muslim League initially Khan, and Syed Ali Zaheer, In December,1946 Maulana
decided to sit out of the all non-League Muslim Abul Kalam Azad was
government. But eventually representatives. Two posts inducted into the cabinet.
joined thus coming of arch remained vacant. But Initially he was not in the
rivals together. afterwards Lord Wavell cabinet
agreed to allot all five
reserved portfolios to the
Muslim League if it agreed
to cooperate, the latter
finally joined.

(Source: https://indianexpress.com/article/explained/september-2-when-indias-interim-govt-was-
formed-in-1946-5959889/ )

IASbaba
Score:
Web: http://ilp.iasbaba.com/
41.00 /
Email: ilp@iasbaba.com
Page 15 200
ILP FRESHERS- 2020 TEST
Exam Title :
41 ...
Email
Contact

QUESTION 15. MjI2MTgwK1BBUlRIK3BhcnRodW1yYWxpeWE1NUBnbWFpbC5jb20rODE0MDA3MTI2OFFVR


NUSU9OIDE0
Which of the following personality is the founder of the Asiatic Society of Mumbai?

a) Sir William Jones


b) James Prinsep
c) Sir Charles Wilkins
d) Sir James Mackintosh
Correct Answer: D
Your Answer: A
Explanation

Solution (d)

Basic Information:

· The Asiatic Society of Mumbai, elected the first woman president Prof Vispi Balaporia in the
215 years of its existence.

· Prof Vispi Balaporia, visiting faculty at Mumbai’s Jai Hind College, an institution where she
was earlier Vice-Principal and Head of the Department of English, will head the institution that
is a treasure house of remarkable historical artifacts.

· The Asiatic Society began its journey in 1804 as the Literary Society of Bombay. It was
founded by Sir James Mackintosh, a Scottish colonial administrator who had a keen interest in
Oriental studies.

· In 1841, the Society started publishing a journal titled Journal of the Bombay Branch of the
Royal Asiatic Society which continues to be published under the name Journal of the Asiatic
Society of Mumbai.

· It was initially started by and for white European men, but in year 1841 the Society began
admitting Indians as members.

· In 1826, the Literary Society became the Mumbai arm of the London-based Royal Asiatic
Society of Great Britain and Ireland and came to be called the Bombay Branch of the Royal
Asiatic Society (BBRAS).

· In 1954, the institution was severed from its London parent and became the Asiatic Society of
Bombay. In 2002, it acquired its present name.

(Source: https://indianexpress.com/article/explained/explained-now-with-first-woman-president-
vispi-balaporia-what-is-asiatic-society-mumbai-5962907/ )

QUESTION 16. MjI2MTgwK1BBUlRIK3BhcnRodW1yYWxpeWE1NUBnbWFpbC5jb20rODE0MDA3MTI2OFFVR


NUSU9OIDE1
Consider the following statements:

IASbaba
Score:
Web: http://ilp.iasbaba.com/
41.00 /
Email: ilp@iasbaba.com
Page 16 200
ILP FRESHERS- 2020 TEST
Exam Title :
41 ...
Email
Contact

1. He was India’s first Vice-President and second President.

2. He sought to spread the correct teachings of Advaita Vedanta.

3. His major work includes Indian Philosophy, article ‘The Ethics of the Bhagavad Gita and Kant’

Which of the personalities is associated with the above mentioned statements?

a) Lal Bahadur Shastri


b) Dr. Sarvepalli Radhakrishnan
c) Varahagiri Venkata Giri
d) Dr. Zakir Husain
Correct Answer: B
Your Answer: B
Explanation

Solution (b)

Basic Information:

· Teacher’s Day is observed annually on September 5, as Dr. Radhakrishnan was born on


September 5,1888.

· When he became the President of India, some of his students and friends requested him to
allow them to celebrate his birthday, on 5 September. He replied, “Instead of celebrating my
birthday, it would be my proud privilege if September 5th is observed as Teachers' Day."

· He was appointed as professor of Philosophy to the King George V Chair of Mental and Moral
Science at the University of Calcutta.

· He was appointed as a Spalding professor of Eastern Religion and Ethics at the University of
Oxford.

· He was the first Indian to be appointed the Vice Chancellor of Banaras Hindu university.

· Radhakrishnan was nominated 16 times for the Nobel Prize in Literature and another 11 times
for the Noble Prize.

· Dr. Sarvepalli Radhakrishnan was appointed as a Knight Bachelor in the year 1931. He was
elected as a Fellow of the British Academy in 1938. He was also awarded the Bharat Ratna in
1954 and the Order of Merit in 1963.

Statement Analysis:

Statement 1 Statement 2 Statement 3

IASbaba
Score:
Web: http://ilp.iasbaba.com/
41.00 /
Email: ilp@iasbaba.com
Page 17 200
ILP FRESHERS- 2020 TEST
Exam Title :
41 ...
Email
Contact

He sought to spread the


correct teachings of
Radhakrishnan was elected Advaita Vedanta among
His literary works included
the first Vice President of those who were ill-informed
– A Source Book in Indian
India in 1952. He was about the true essence of
Philosophy, The World
president of India from the Hindu religion. He
Treasury of Modern
1962 to 1967, after prepared a thesis on the
Religious, Idealist View of
Rajendra Prasad (first ethics of the Vedanta,
life, Religion and Culture,
President of India, in office which was intended to be a
etc.
from 1950 to 1962.) reply to the charge that the
Vedanta system had no
room for ethics.

(Source: https://theprint.in/theprint-profile/sarvepalli-radhakrishnan-the-president-who-
defended-hinduism-against-western-criticism/222458/ and https://
economictimes.indiatimes.com/news/politics-and-nation/philosopher-teacher-president-
remembering-dr-s-radhakrishnan/proficient-statesman/slideshow/60374997.cms

QUESTION 17. MjI2MTgwK1BBUlRIK3BhcnRodW1yYWxpeWE1NUBnbWFpbC5jb20rODE0MDA3MTI2OFFVR


NUSU9OIDE2
Which of the following event did not occur in the aftermath of withdrawal of Civil Disobedience
Movement?

a) Jayaprakash Narayan, Achhut Patwardhan, Asoke Mehta, Yusuf Mehrali, Narendra Dev and
Minoo Masani-formed the Swatantra Party (CSP).
b) Internal differences emerged in Indian National Congress over the future strategy of the
national movement.
c) Gandhi announced his resignation from the Congress to serve it better in thought, word
and deed.
d) Both option (a) and (c)
Correct Answer: A
Your Answer: Unanswered
Explanation

Solution (a)

Basic Information:

· The INC officially suspended the CDM in May 1933 and withdrew it in May 1934.

· The CSP formed in 1934 operated within INC and tried to change its orientation towards a
socialist programme as well as contain the dominance of the conservative 'right' wingers.

· At the Lucknow Congress in 1936 two issues, i.e., council entry and office acceptance came to
a head

IASbaba
Score:
Web: http://ilp.iasbaba.com/
41.00 /
Email: ilp@iasbaba.com
Page 18 200
ILP FRESHERS- 2020 TEST
Exam Title :
41 ...
Email
Contact

· The AICC meeting (August 1936) in Bombay decided in favour of contesting the election, but
postponed the decision on office acceptance until the election was over.

· The AICC sanctioned office acceptance in March, 1937 by overriding the objections of Nehru
and other CSP leaders.

· The formation of the All India Kisan Sabha (AIKS) at the Lucknow session of the Congress in
April 1936, with Sahajanand Saraswati elected as its first president.

Statement Analysis:

Option a Option b Option c

Incorrect Correct Correct

A large section of the


The socialists and other There was a two-stage intelligentsia with INC
leftist elements-the most debate on the future favoured parliamentary
important of them being strategy of the national
Jayaprakash Narayan, movement during the phase Politics, another section
Achhut Patwardhan, Asoke of non-mass struggle – was estranged because of
Mehta, Yusuf Mehrali, Constructive work of Gandhi's emphasis on the
Narendra Dev and Minoo Gandhi and Council Entry.
Masani-formed in May 1934 In 1937, differences spinning wheel as the
the Congress Socialist emerged over the question "second lung of the nation".
Party (CSP). C of office acceptance in the Nehru also had differences
Rajagopalachari founded context with Gandhi over Struggle -
the Swatantra Party in Truce-Struggle (S-T-S)
1959. of provincial elections
strategy

(Source: From Plassey to Partition: A History of Modern India)

QUESTION 18. MjI2MTgwK1BBUlRIK3BhcnRodW1yYWxpeWE1NUBnbWFpbC5jb20rODE0MDA3MTI2OFFVR


NUSU9OIDE3
While launching which of the following mass movement, Gandhi called it "the biggest fight in
my life"?

a) Civil Disobedience Movement


b) Fasting against Communal Accord
c) Non-Cooperation Movement
d) Quit India Movement
Correct Answer: D
Your Answer: B
Explanation

IASbaba
Score:
Web: http://ilp.iasbaba.com/
41.00 /
Email: ilp@iasbaba.com
Page 19 200
ILP FRESHERS- 2020 TEST
Exam Title :
41 ...
Email
Contact

Solution (d)

Basic Information:

· At the opening day of the All India Congress Committee (AICC) at the Gowalia tank Maidan in
Mumbai on 7 th August, 1942; Gandhi said, "I stick to the principle of non-violence as I did
before. If you are tired of it then you need not come with me. At the time when I am about to
launch the biggest fight in my life there can be no hatred for the British in my heart"

· The "Quit India" resolution, adopted by the AICC in Bombay on 8 August 1942, proposed to
begin this mass civil disobedience under Gandhi's direction

· He briskly set aside all opposition from within the Congress against direct action, coming
mainly from Nehru and Rajagopalachari, and prepared the party for the final struggle, "the
biggest fight in my life".

· 77th anniversary of Quit India movement was observed on August 8th, 2019. Every year 8
August is celebrated in India as August Kranti Din.

(Source: From Plassey to Partition: A History of Modern India )

QUESTION 19. MjI2MTgwK1BBUlRIK3BhcnRodW1yYWxpeWE1NUBnbWFpbC5jb20rODE0MDA3MTI2OFFVR


NUSU9OIDE4
Consider the following statements with reference to Lal Bahadur Shastri:

1. He became a life member of the Servants of the People Society (Lok Sevak Mandal), founded
by Lala Lajpat Rai.

2. He participated in the non-cooperation movement and the Salt Satyagraha.

3. During Nehru’s illness, he was Defence Minister of India.

Which of the statements given above is/are correct?

a) 1 and 2 Only
b) 1 and 3 Only
c) 3 Only
d) 2 and 3 Only
Correct Answer: A
Your Answer: Unanswered
Explanation

Solution (a)

Basic Information:

· Lal Bahadur Shastri was born on 2nd October, 1904 at Mughalsarai, Varanasi, Uttar Pradesh.

IASbaba
Score:
Web: http://ilp.iasbaba.com/
41.00 /
Email: ilp@iasbaba.com
Page 20 200
ILP FRESHERS- 2020 TEST
Exam Title :
41 ...
Email
Contact

· He was greatly impressed by Mahatma Gandhi’s denunciation of Indian Princes for their
support of British rule in India.

· Lal Bahadur Shastri joined the Kashi Vidya Peeth in Varanasi, one of the many national
institutions set up in defiance of the British rule.

· ‘Shastri’ was the bachelor’s degree awarded to him by the Vidya Peeth but has stuck in the
minds of the people as part of his name.

· He accepted a spinning wheel and a few yards of handspun cloth as dowry.

· When the Congress Government was formed in 1946, he was appointed Parliamentary
Secretary in his home State of Uttar Pradesh and soon rose to the position of Home Minister.

· He resigned his post as Minister for Railways because he felt responsible for a railway
accident in which many lives were lost. The unprecedented gesture was greatly appreciated by
Parliament and the country.

· He was the second Prime Minister of Independent India succeeding Nehru.

· He promoted the White Revolution and gave slogan – “Jai Jawan Jai Kisan”

· He was awarded the Bharat Ratna the India’s highest civilian award posthumously in 1966.

· He suffered a heart attack while in Tashkent and passed away there.

Statement Analysis:

Statement 1 Statement 2 Statement 3

Correct Correct Incorrect

Lal Bahadur Shastri was


He held several portfolios
sixteen when Gandhiji
in the Union Cabinet –
In Lok Seva mandal he called upon his countrymen
Minister for Railways;
started to work for the to join the Non-Cooperation
Minister for Transport and
upliftment of backward Movement. He threw
Communications; Minister
classes, and later he himself into the Salt-
for Commerce and
became the President of Satyagraha led many
Industry; Home Minister;
that Society. defiant campaigns and
and during Nehru’s illness
spent a total of seven years
Minister without portfolio.
in British jails.

(Source: https://www.pmindia.gov.in/en/former_pm/shri-lal-bahadur-shastri/ )

QUESTION 20. MjI2MTgwK1BBUlRIK3BhcnRodW1yYWxpeWE1NUBnbWFpbC5jb20rODE0MDA3MTI2OFFVR


NUSU9OIDE5

IASbaba
Score:
Web: http://ilp.iasbaba.com/
41.00 /
Email: ilp@iasbaba.com
Page 21 200
ILP FRESHERS- 2020 TEST
Exam Title :
41 ...
Email
Contact

What was one of the objectives of Home Rule Movement?

a) To disobey British laws and make it difficult to govern.


b) Complete transfer of power to Indians.
c) Obtain Dominion Status within British Empire.
d) Boycott of foreign goods
Correct Answer: C
Your Answer: C
Explanation

Solution (c)

Basic Information:

· The Home Rule Movement was the Indian response to the First World War.

· The Indian Home Rule Leagues were organized on the lines of the Irish Home Rule Leagues
and they represented the emergence of a new trend of aggressive politics.

· Annie Besant and Tilak were the pioneers of this new trend.

· The factors like - The Moderates were disillusionment with the Morley-Minto reforms, People
were feeling the burden of wartime miseries, etc. led the ground for the Home Rule movement.

· The basic objectives were; the national demand for self-government (Home Rule), and to
obtain the status of Dominion within the British empire as enjoyed by Australia, Canada, South
Africa, New Zealand and Newfoundland at that time.

· Two Home Rule Leagues were established, one by B.G TIlak at Pune in April, 1916 and was
restricted to Maharashtra (excluding Bombay city), Karnataka, Central Provinces and Berar.
The other league by Annie Besant at Madras in September, 1916 covered the rest of India
(including Bombay city).

· The Home Rule agitation was later joined by Motilal Nehru, JawaharlalNehru, Bhulabhai
Desai, Chittaranjan Das, Madan Mohan Malaviya, Mohammad Ali jinnah, Tej Bahadur Sapru and
Lala Lajpat Rai.

(Source: Spectrum)

QUESTION 21. MjI2MTgwK1BBUlRIK3BhcnRodW1yYWxpeWE1NUBnbWFpbC5jb20rODE0MDA3MTI2OFFVR


NUSU9OIDIw
Which of the following is the correct reason for the resignation of Congress ministers in the
seven provinces in 1939?

a) Rising socialist and communist influence in the country making it difficult to work for
Congress ministers.
b) Widespread communal tensions in the country.

IASbaba
Score:
Web: http://ilp.iasbaba.com/
41.00 /
Email: ilp@iasbaba.com
Page 22 200
ILP FRESHERS- 2020 TEST
Exam Title :
41 ...
Email
Contact

c) In protest against Viceroy declaring India as party to 2 nd World War without consulting
Indians
d) Rejection of Congress proposal to form Constituent Assembly.
Correct Answer: C
Your Answer: C
Explanation

Solution (c)

Basic Information:

· The Congress contested 716 out of 1161 seats. It got a majority in all provinces, except in
Bengal, Assam, Punjab, Sindh and NWFP, and emerged as the largest party in Bengal, Assam
and NWFP.

· Congress ministries were formed in Bombay, Madras, Central Provinces, Orissa, United
Provinces, Bihar and later in NWFP and Assam also.

· On September 1, 1939: Germany attacks Poland, Second World War started.

· On September 3, 1939: Britain declares war against Germany and declares India's support for
the war without consulting Indian opinion.

· The Congress Working Committee (CWC) resolution condemned Fascist aggression. But also
argued that: India could not be party to a war being fought ostensibly for democratic freedom,
while that freedom was being denied to India; if Britain was fighting for democracy and
freedom, it should prove it by ending imperialism in its colonies and establishing full democracy
in India.

· The Congress leadership wanted to give every chance to the viceroy and the British
Government.

· The Government's response was entirely negative. Linlithgow, in his statement (October 17,
1939), tried to use the Muslim League and the princes against the Congress.

· On October 23, 1939, the CWC meeting decided not to support the war, and called upon the
Congress ministries to resign in the provinces

(Source: Spectrum)

QUESTION 22. MjI2MTgwK1BBUlRIK3BhcnRodW1yYWxpeWE1NUBnbWFpbC5jb20rODE0MDA3MTI2OFFVR


NUSU9OIDIx
Consider the following statements with reference to Ishwar Chandra Vidyasagar:

1. He joined the Fort William College as the Head Pandit in the Sanskrit department.

2. He was appointed principal of Sanskrit College in 1851.

3. His major contribution was to persuade British government to enact a law abolishing the
custom of Sati in 1829.

IASbaba
Score:
Web: http://ilp.iasbaba.com/
41.00 /
Email: ilp@iasbaba.com
Page 23 200
ILP FRESHERS- 2020 TEST
Exam Title :
41 ...
Email
Contact

Which of the statements given above is/are correct?

a) 1 and 2 Only
b) 2 and 3 Only
c) 1 and 3 Only
d) 1, 2 and 3 Only
Correct Answer: A
Your Answer: D
Explanation

Solution (a)

Basic Information:

· Ishwar Chandra Bandopadhyaya was born in Birsingha village of Midnapore district in Bengal
on September 26, 1820.

· He learned Vedanta, Vyakaran, Literature, Rhetoric’s, Smriti and Ethics in Sanskrit College
during 1829 to 1841.

· He took part in a competition testing knowledge in Sanskrit in 1839 and earned the title of
‘Vidyasagar’ meaning Ocean of Knowledge. The same year Ishwar Chandra Vidyasagar
successfully cleared his Law examination.

· He wrote two books ‘Upakramonika’ and ‘Byakaran Koumudi’, interpreting complex notions of
Sanskrit grammar in easy legible Bengali language.

· He was an ardent advocate of women education. He supported John Elliot Drinkwater Bethune
to establish the first permanent girls’ school in India, the Bethune School, on May 7, 1849.

· His lasting legacy remains with ‘Borno Porichoy’, an elementary level book for learning
Bengali alphabets.

· He campaigned against polygamy.

· On October 14, 1855, Vidyasagar presented a petition to the Government of India praying for
early passing a law to remove all obstacles to the marriage of Hindu widows and to declare the
issue of all such marriages to be legitimate.

Statement Analysis:

Statement 1 Statement 2 Statement 3

Correct Correct Incorrect

IASbaba
Score:
Web: http://ilp.iasbaba.com/
41.00 /
Email: ilp@iasbaba.com
Page 24 200
ILP FRESHERS- 2020 TEST
Exam Title :
41 ...
Email
Contact

He became Professor in
Sanskrit College on the
request of the college
authorities but imposed a
condition that he be
Rajaram Mohan Roy played
In 1841, at the age of allowed to redesign the
crucial role in passing law
twenty one, Ishwar system. He became
abolishing the custom of
Chandra joined the Fort Principal of Sanskrit
Sati in 1829. Vidyasagar
William College as the College in 1851. In 1855,
played major role in
Head Pandit in the Sanskrit he assumed the
Widows' Remarriage Act,
department. responsibilities as a special
1856
inspector of schools with
additional charges and
travelled to remote villages
in Bengal to oversee the
quality of education.

(Source: https://indianexpress.com/article/explained/vidyasagar-history-of-indian-social-reform-
kolkata-violence-5729699/ )

QUESTION 23. MjI2MTgwK1BBUlRIK3BhcnRodW1yYWxpeWE1NUBnbWFpbC5jb20rODE0MDA3MTI2OFFVR


NUSU9OIDIy
With reference to the period of Indian freedom struggle, which of the following was/were
recommended by the Nehru report?

1. Complete Independence for India

2. Joint electorates for reservation of seats for minorities.

3. Provision of fundamental rights for the people of India in the Constitution.

Which of the statements given above is/are correct?

a) 2 Only
b) 2 and 3 Only
c) 3 Only
d) 1, 2 and 3 Only
Correct Answer: B
Your Answer: D
Explanation

Solution (b)

Basic Information:

· In 1927 Conservative Government of Britain appointed the Indian Statutory Commission,


popularly known as the Simon Commission.

IASbaba
Score:
Web: http://ilp.iasbaba.com/
41.00 /
Email: ilp@iasbaba.com
Page 25 200
ILP FRESHERS- 2020 TEST
Exam Title :
41 ...
Email
Contact

· No Indian was considered fit to serve on a body that claimed the right to decide the political
future of India.

· Lord Birkenhead, the Conservative Secretary of State questioned ability of Indians to


formulate a concrete scheme of constitutional reforms which had the support of wide sections
of Indian political opinion.

· As a response, All-Parties Conference were held in February, May and August 1928 to finalize
a scheme which popularly came to be known as the Nehru Report after Motilal Nehru, its
principal author.

· Nehru Report (1928) was first Indian effort to draft constitutional scheme.

· Muslim league and even Jinnah would not give up the demand for reservation of seats for
Muslims especially in Muslim majority provinces.

· With no further concessions were forthcoming, Jinnah withdrew his support to the report and
went ahead to propose his famous ‘Fourteen Points’

· Other recommendations include: A federal form of government with residual powers with the
centre, ministry would be responsible to the legislature, a proposal for the creation of a
Supreme Court, provinces would be created along linguistic lines, etc.

Statement Analysis:

Statement 1 Statement 2 Statement 3

Incorrect Correct Correct

It also rejected the


principle of separate
communal electorates on 19 fundamental rights like
which previous The Report also
This report defined constitutional reforms had recommended universal
Dominion Status as the been based. Seats would be adult suffrage, equal rights
form of government desired reserved for Muslims at the for women, freedom to form
by India. Centre and in provinces in unions, and dissociation of
which they were in a the state from religion in
minority, but not in those any form, etc.
where they had a numerical
majority.

(Source: India’s Struggle for Independence)

QUESTION 24. MjI2MTgwK1BBUlRIK3BhcnRodW1yYWxpeWE1NUBnbWFpbC5jb20rODE0MDA3MTI2OFFVR


NUSU9OIDIz

IASbaba
Score:
Web: http://ilp.iasbaba.com/
41.00 /
Email: ilp@iasbaba.com
Page 26 200
ILP FRESHERS- 2020 TEST
Exam Title :
41 ...
Email
Contact

Consider the following statements:

1. In 1920, many prominent freedom fighters including Vithalbhai Patel, Mahatma Gandhi and
Bal Gangadhar Tilak demanded his release from jail.

2. He wrote the book ‘Hindutva: who is hindu?

3. Under his leadership, Hindu Mahasabha collaborated with Muslim league to form
government in provinces like Sindh, Bengal and North West Frontier Province.

Which of the following nationalist leader of India’s freedom struggle is associated with above
statements?

a) Keshav Baliram Hedgewar


b) Madan Mohan Malaviya
c) Lala Lajpat Rai
d) Vinayak Damodar Savarkar
Correct Answer: D
Your Answer: C
Explanation

Solution (d)

Basic Information:

· Vinayak Damodar Savarkar aka Swatantryaveer Savarkar known as the father of Hindu
rashtravad (Hindu nationalism), but also represents Maharashtra’s glorious tradition of social
and religious reformers.

· Savarkar began his political activities as a high school student and continued to do so at
Fergusson College in Pune.

· He and his brother founded a secret society called Abhinav Bharat Society.

· He went to the United Kingdom for his law studies, he involved himself with organizations
such India House and the Free India Society.

· He was charged under abetment to murder of Nashik Collector Jackson and waging a
conspiracy under Indian penal code 121-A against the King emperor.

· Aged 28, he was convicted and sentenced to 50-years imprisonment and transported on 4 July
1911 to the infamous Cellular Jail in the Andaman and Nicobar Islands.

· In his book 'The History of the War of Indian Independence', became one of the first writers to
call for India's first war for independence against the British.

· Being a passionate promoter of Hindutva, he never was a cow worshipper.

· When the Congress launched the Quit India movement in 1942, Savarkar criticised it and
asked Hindus to stay active in the war effort and not disobey the government.

IASbaba
Score:
Web: http://ilp.iasbaba.com/
41.00 /
Email: ilp@iasbaba.com
Page 27 200
ILP FRESHERS- 2020 TEST
Exam Title :
41 ...
Email
Contact

· In 1964, he felt that his goal of independence India is achieved and he declared his wish to
attain Samadhi. He started a hunger strike on February 1, 1966, and passed away on February
26, 1966.

(Source: Wikipedia )

QUESTION 25. MjI2MTgwK1BBUlRIK3BhcnRodW1yYWxpeWE1NUBnbWFpbC5jb20rODE0MDA3MTI2OFFVR


NUSU9OIDI0
Even when there are doubts over pre-planning of 1857 revolt, but there are evidences of
planning once the revolt started. Which of the following statement are planning efforts taken by
rebels once the revolt started?

1. A letter was addressed to the rulers of all the neighbouring states asking their support and
inviting them to participate.

2. In Delhi, a court of administrators was established which was responsible for all matters of
state.

3. Coins were struck and orders were issued in the name of Mughal emperor to provide
legitimacy to movement.

Select the correct code:

a) 2 Only
b) 2 and 3 Only
c) 3 Only
d) 1, 2 and 3 Only
Correct Answer: D
Your Answer: D
Explanation

Solution (d)

Basic Information:

· Was the revolt an organized and methodically planned Revolt or a spontaneous insurrection, is
still a matter of debate.

· The absence of any reliable account left behind by the rebels make it difficult to clear the
confusion.

· When the sepoys arrived from Meerut, Bahadur Shah seems to have been taken by surprise
and promptly conveyed the news to the Lt.Governor at Agra.

· Rani Lakshmibhai of Jhansi who took quite some time before openly joining the rebels.

· The message conveyed by the circulation of chappatis and lotus flowers is also uncertain.

IASbaba
Score:
Web: http://ilp.iasbaba.com/
41.00 /
Email: ilp@iasbaba.com
Page 28 200
ILP FRESHERS- 2020 TEST
Exam Title :
41 ...
Email
Contact

· But even if there was no planning and organization before the revolt, it was important that it
was done, once it started

· Immediately after the capture of Delhi a letter was addressed to the rulers of all the
neighbouring states and of Rajasthan soliciting their support and inviting them to participate.

· In Delhi, a court of administrators was established which was responsible for all matters of
state. The court consisted of ten members, six from the army and four from the civilian
departments. All decisions were taken by a majority vote.

· The court conducted the affairs of the state in the name of the Emperor. ‘The Government at
Delhi.

· Khan Bahadur Khan conducted the administration in the name of the Mughal Emperor.

(Source: India’s Struggle for Independence)

QUESTION 26. MjI2MTgwK1BBUlRIK3BhcnRodW1yYWxpeWE1NUBnbWFpbC5jb20rODE0MDA3MTI2OFFVR


NUSU9OIDI1
Which of the following political philosopher in his massive work ‘A History of British India’
divided Indian history into three periods – Hindu, Muslim and British?

a) Thomas Babington Macaulay


b) Jeremy Bentham
c) James Mill
d) John Stuart Mill
Correct Answer: C
Your Answer: Unanswered
Explanation

Solution (c)

Basic Information:

· In 1817, James Mill, a Scottish economist and political philosopher, published a massive three-
volume work, A History of British India. In this he divided Indian history into three periods –
Hindu, Muslim and British. On the basis of this work, Mills secured the post of an examiner at
the imperial East India Company

· Mill thought that all Asian societies were at a lower level of civilisation than Europe.
According to his telling of history, before the British came to India, Hindu and Muslim despots
ruled the country.

· Mill felt British rule could civilise India.

· Mill, in fact, suggested that the British should conquer all the territories in India to ensure the
enlightenment and happiness of the Indian people. For India was not capable of progress
without British help.

IASbaba
Score:
Web: http://ilp.iasbaba.com/
41.00 /
Email: ilp@iasbaba.com
Page 29 200
ILP FRESHERS- 2020 TEST
Exam Title :
41 ...
Email
Contact

· Historians also have divided Indian history into ‘ancient’, ‘medieval’ and ‘modern’.

(Source: Our Past – III Part – 1, Class VIII NCERT)

QUESTION 27. MjI2MTgwK1BBUlRIK3BhcnRodW1yYWxpeWE1NUBnbWFpbC5jb20rODE0MDA3MTI2OFFVR


NUSU9OIDI2
In 1600, the East India Company acquired a charter from the ruler of England, Queen Elizabeth
I, granting it the complete monopoly to trade with the East. This monopoly was completely
taken away by which of the following British act?

a) Charter Act of 1773


b) Charter Act of 1813
c) Charter Act of 1833
d) Charter Act of 1853
Correct Answer: C
Your Answer: C
Explanation

Solution (c)

Basic Information:

· The Charter Act of 1833 ended the activities of EIC as a commercial body and made it a purely
administrative body.

· The Charter act of 1813 ended the monopoly of the East India Company in India, however the
company’s monopoly in trade with China and trade in tea was allowed.

· Under it the company lost its trade monopoly with China and trade in tea.

· The Charter Act of 1833 gave another lease of life for twenty years to the East India Company
which was to administer the Indian territories “in trust for His Majesty, his heirs and
successors”.

· It centralised the Indian administration and designated the Governor-General of Bengal as


Governor-General of India. Thus, Lord William Bentinck became the “First Governor General of
British India”.

· It deprived the Governors of Bombay and Madras of their legislative powers.

· The act also directed the governor general in council to adopt measures to mitigate the state
of slavery in India.

(Source: Our Past – III Part – 1, Class VIII NCERT)

QUESTION 28. MjI2MTgwK1BBUlRIK3BhcnRodW1yYWxpeWE1NUBnbWFpbC5jb20rODE0MDA3MTI2OFFVR


NUSU9OIDI3

IASbaba
Score:
Web: http://ilp.iasbaba.com/
41.00 /
Email: ilp@iasbaba.com
Page 30 200
ILP FRESHERS- 2020 TEST
Exam Title :
41 ...
Email
Contact

Saadat Ali Khan, the Nawab of Awadh was forced to give over half of his territory to the
Company in 1801 because of:

a) His failure to pay for subsidiary forces that the Company was supposed to maintain for the
purpose of protection.
b) Allegations of misgovernment on the Nawab of Awadh.
c) The allegations of treachery and disloyalty to British.
d) None of the above.
Correct Answer: A
Your Answer: Unanswered
Explanation

Solution (a)

Basic Information:

· The Company forced the states into a “subsidiary alliance”. According to the terms of this
alliance, Indian rulers were not allowed to have their independent armed forces.

· They were to be protected by the Company, but had to pay for the “subsidiary forces” that the
Company was supposed to maintain for the purpose of this protection.

· If the Indian rulers failed to make the payment, then part of their territory was taken away as
penalty.

· For example, when Richard Wellesley was Governor General (1798-1805), the Nawab of
Awadh was forced to give over half of his territory to the Company in 1801, as he failed to pay
for the “subsidiary forces”.

· Hyderabad was also forced to cede territories of Northern Sarkars on similar grounds.

· In 1856, Lord Dalhousie annexed remaining half of Awadh under the allegations of
misgovernment. Wajid Ali Shah, the then Nawab, was imprisoned, and then exiled by the
Company to Calcutta (Bengal).

(Source: Our Past – III Part – 1, Class VIII NCERT)

QUESTION 29. MjI2MTgwK1BBUlRIK3BhcnRodW1yYWxpeWE1NUBnbWFpbC5jb20rODE0MDA3MTI2OFFVR


NUSU9OIDI4
Who among the following attended all the three Round Table Conferences:

1. Madan Mohan Malaviya

2. B. R. Ambedkar

3. Mahatma Gandhi

4. Tej Bahadur Sapru

Select the correct code:

IASbaba
Score:
Web: http://ilp.iasbaba.com/
41.00 /
Email: ilp@iasbaba.com
Page 31 200
ILP FRESHERS- 2020 TEST
Exam Title :
41 ...
Email
Contact

a) 1 and 3 Only
b) 2 and 3only
c) 2 and 4 Only
d) 1, 2 and 4 Only
Correct Answer: C
Your Answer: D
Explanation

Solution (c)

Basic Information:

· Tej Bahadur Sapru and Dr. B R Ambedkar, Begum Jahanara Shahnawaz (Women
representation), even Muslim league leader Aga Khan III attended all the three Round Table
Conferences.

· The First Round Table Conference officially inaugurated by His Majesty George V on
November 12, 1930 in Royal Gallery House of Lords at London and chaired by the Prime
Minister. Ramsay MacDonald. It was attended by Muslim League members like Aga Khan III,
Jinnah, etc.; Liberals represented by Tej Bahadur Sapru; Depressed Classes by B. R. Ambedkar;
and others.

· The Second Round Table Conference opened on 7 September, 1931 was attended by M K
Gandhi as sole representative of Congress; Liberals represented by Tej Bahadur Sapru;
Depressed Classes by B. R. Ambedkar, Muslim League leader Aga Khan III; Madan Mohan
Malaviya, etc.

· The Third Round Table Conference assembled on November 17, 1932 was attended by Aga
Khan III, B. R. Ambedkar (Depressed Classes), Begum Jahanara Shahnawaz (Women), A. P.
Patro, Tej Bahadur Sapru, etc.

(Source: Wikipedia)

QUESTION 30. MjI2MTgwK1BBUlRIK3BhcnRodW1yYWxpeWE1NUBnbWFpbC5jb20rODE0MDA3MTI2OFFVR


NUSU9OIDI5
The Permanent Settlement introduced in 1793 to ensure regular flow of revenue to British
however created many problems. In this context consider the following statements:

1. The zamindars invested heavily in land improvement to benefit from increase production of
land.

2. In case of non-payment of rent by peasant, he was evicted from land.

3. Zamindars who failed to pay the revenue to British lost their zamindari rights.

Which of the statements given above correctly highlight the problems created by the Permanent
Settlement?

a) 2 Only

IASbaba
Score:
Web: http://ilp.iasbaba.com/
41.00 /
Email: ilp@iasbaba.com
Page 32 200
ILP FRESHERS- 2020 TEST
Exam Title :
41 ...
Email
Contact

b) 2 and 3 Only
c) 1 and 3 Only
d) 1, 2 and 3 Only
Correct Answer: B
Your Answer: Unanswered
Explanation

Solution (b)

Basic Information:

· The Permanent Settlement was introduced in 1793 by Lord Cornwallis.

· The rajas and taluqdars were recognised as zamindars and were responsible to collet rent
from the peasants and pay revenue to the company.

· The amount to be paid was permanently fixed.

· It was felt that this would ensure a regular flow of revenue into the Company’s coffers and at
the same time encourage the zamindars to invest in improving the land.

· But soon Company officials realised the problems created by the Settlement

Statement Analysis:

Statement 1 Statement 2 Statement 3

Incorrect Correct Correct

The villages, cultivators


found the system extremely
The revenue that has been
oppressive. The rent to be
fixed permanently was so
The zamindars were not paid was high and his right
high that the zamindars
investing in land on the land was insecure.
found it difficult to pay. So
improvement. Burden of repaying loan,
failure to pay revenue
high rent caused non-
caused loss of zamindari
payment of rent and
eviction from land

(Source: Our Past – III Part – 1, Class VIII NCERT)

QUESTION 31. MjI2MTgwK1BBUlRIK3BhcnRodW1yYWxpeWE1NUBnbWFpbC5jb20rODE0MDA3MTI2OFFVR


USU9OIDMw
Which of the following statement(s) is/are correct regarding the main systems of indigo
cultivation under British India?

IASbaba
Score:
Web: http://ilp.iasbaba.com/
41.00 /
Email: ilp@iasbaba.com
Page 33 200
ILP FRESHERS- 2020 TEST
Exam Title :
41 ...
Email
Contact

1. Under ‘nij’ system the planter produced indigo in lands that he directly controlled.

2. The planters signed a contract, an agreement (satta) with the ryots to cultivate indigo under
‘ryoti’ system.

Select the correct code:

a) 1 Only
b) 2 Only
c) Both 1 and 2
d) Neither 1 nor 2
Correct Answer: C
Your Answer: C
Explanation

Solution (c)

Basic Information:

· Faced with the rising demand for indigo in Europe, the Company in India looked for ways to
expand the area under indigo cultivation.

· Attracted by the prospect of high profits, numerous Scotsmen and Englishmen came to India
and became planters.

· There were two main systems of indigo cultivation – nij and ryoti.

· Within the system of nij cultivation, the planter produced indigo in lands that he directly
controlled. He either bought the land or rented it from other zamindars and produced indigo by
directly employing hired labourers.

· Under the ryoti system, the planters forced the ryots to sign a contract, an agreement (satta).
At times they pressurised the village headmen to sign the contract on behalf of the ryots. Those
who signed the contract got cash advances from the planters at low rates of interest to produce
indigo.

(Source: Our Past – III Part – 1, Class VIII NCERT)

QUESTION 32. MjI2MTgwK1BBUlRIK3BhcnRodW1yYWxpeWE1NUBnbWFpbC5jb20rODE0MDA3MTI2OFFVR


NUSU9OIDMx
Consider the following statements with reference to the Indigo revolt (1859-61):

1. Peasants were forced to grow indigo on 3/20 of the total land under Tinkathia system.

2. The Indigo Commission set up by the government to enquire into system of indigo
production, criticised planters for using coercive methods with indigo cultivators.

3. After the Indigo revolt the planters shifted their indigo operations from Bihar to Bengal

Which of the statements given above is/are incorrect?

IASbaba
Score:
Web: http://ilp.iasbaba.com/
41.00 /
Email: ilp@iasbaba.com
Page 34 200
ILP FRESHERS- 2020 TEST
Exam Title :
41 ...
Email
Contact

a) 1 Only
b) 2 and 3 Only
c) 1 and 3 Only
d) 1, 2 and 3 Only
Correct Answer: C
Your Answer: Unanswered
Explanation

Solution (c)

Basic Information:

· The Indigo Rebellion (Neel Bidroho) took place in Bengal in 1859-60 and was a revolt by the
farmers against British planters who had forced them to grow indigo under terms that were
greatly unfavourable to the farmers.

· The farmers were given dvanced loans for this purpose. Once the farmers took loans, they
could never repay it due to the high rates of interest.

· The government always supported the planters who enjoyed many privileges and judicial
immunities.

· The indigo farmers revolted in the Nadia district of Bengal by refusing to grow indigo.

· The farmers were led by the Biswas brothers of Nadia, Rafiq Mondal of Malda and Kader
Molla of Pabna. The revolt also received support from many zamindars notably Ramrattan
Mullick of Narail.

· The play Nil Darpan (The Mirror of Indigo) by Dinabandhu Mitra written in 1858 – 59
portrayed the farmers’ situation accurately.

· Worried by the rebellion, the government brought in the military to protect the planters from
assault, and set up the Indigo Commission to enquire into the system of indigo production

· After the revolt, indigo production collapsed in Bengal. But the planters now shifted their
operation to Bihar.

· When Mahatma Gandhi returned from South Africa, a peasant from Bihar persuaded him visit
Champaran and see the plight of the indigo cultivators there. Mahatma Gandhi’s visit in 1917
marked the beginning of the Champaran movement against the indigo planters.

Statement Analysis:

Statement 1 Statement 2 Statement 3

Incorrect Correct Incorrect

IASbaba
Score:
Web: http://ilp.iasbaba.com/
41.00 /
Email: ilp@iasbaba.com
Page 35 200
ILP FRESHERS- 2020 TEST
Exam Title :
41 ...
Email
Contact

Indigo Commission
criticised planters, it
declared that indigo Indigo revolt took place in
Tinkathia system is related
production was not Bengal, and after revolt
to Champaran Satyagraha.
profitable for peasants. It planters shifted to Bihar.
asked the ryots to fulfil
their existing contracts.

(Source: Our Past – III Part – 1, Class VIII NCERT

QUESTION 33. MjI2MTgwK1BBUlRIK3BhcnRodW1yYWxpeWE1NUBnbWFpbC5jb20rODE0MDA3MTI2OFFVR


NUSU9OIDMy
Consider the following pairs with reference to Revolt of 1857:

Leader Region

1. Rani Avantibai Lodhi Kanpur

2. Ahmadullah Shah Faizabad

3. Begum Hazrat Mahal Lucknow

Which of the pairs given above is/are incorrect?

a) 1 Only
b) 2 and 3 Only
c) 1 and 3 Only
d) 1, 2 and 3 Only
Correct Answer: A
Your Answer: A
Explanation

Solution (a)

Basic Information:

· On May 10, soldiers from 3rd Native Cavalry released their imprisoned comrades, killed their
officers and unfurled the banner of revolt. They set off for Delhi after sunset thus the rise of
1857 revolt.

· Regiment after regiment mutinied and took off to join other troops at nodal points of revolt
like Delhi, Kanpur and Lucknow.

· In the Mandla region of Madhya Pradesh Rani Avantibai Lodhi of Ramgarh raised and led an
army of four thousand against the British.

· Nana Saheb the adopted son of the late Peshwa Bajirao II who lived near Kanpur, gathered
armed forces and expelled the British garrison from the city and proclaimed himself Peshwa.

IASbaba
Score:
Web: http://ilp.iasbaba.com/
41.00 /
Email: ilp@iasbaba.com
Page 36 200
ILP FRESHERS- 2020 TEST
Exam Title :
41 ...
Email
Contact

· In Lucknow Birjis Qadir, the son of the deposed Nawab wajid Ali was proclaimed as new
Nawab under his mother Begum Hazrat Mahal. She took active part in the uprising.

· Many new leaders came up like Ahmadullah Shah a maulavi fromFaizabad, prophesied that the
rule of the British would come to an end soon. He raised a huge force of supporters.

· In Jhansi, Rani Lakshimibai joined the rebel sepoys and fought along with Tatya Tope.

· Bakht Khan a soldier from Bareilly, took charge of a large force of fighters who came to Delhi.

· Kunwar Singh, an old zamindar from Bihar, joined the rebel sepoys and battled with the
British.

(Source: Our Past – III Part – 1, Class VIII NCERT) )

QUESTION 34. MjI2MTgwK1BBUlRIK3BhcnRodW1yYWxpeWE1NUBnbWFpbC5jb20rODE0MDA3MTI2OFFVR


NUSU9OIDMz
Which of the following architect(s) was/were called to design New Delhi and its buildings?

a) Edward Lutyens
b) Herbert Baker
c) Robert Tor Russel
d) Both the options (a) and (b)
Correct Answer: D
Your Answer: D
Explanation

Solution (d)

Basic Information:

· In 1911, when King George V was crowned in England, a Durbar was held in Delhi to
celebrate the occasion. The decision to shift capital of India from Calcutta to Delhi was
announced at this Durbar.

· New Delhi was constructed as a 10 square mile city on Raisina Hill, south of the existing city.

· Two architects, Edward Lutyens and Herbert Baker, where callrd on to design New Delhi and
its buildings.

· The government complex in New Delhi consisted of a two-mile avenue, Kingsway (now
Rajpath), that led to the Viceroy’s Palace (now Rashtrapati Bhawan), with the Secretariat
buildings on the either sides of the avenue.

· The features of these buildings were borrowed from different periods of India’s imperial
history, but overall look was Classical Greece.

· The central dome of the Viceroy’s palace was copied from the Buddhist stupa at Sanchi, and
the red sandstone and carved screens or jalis were borrowed from Mughal architecture.

IASbaba
Score:
Web: http://ilp.iasbaba.com/
41.00 /
Email: ilp@iasbaba.com
Page 37 200
ILP FRESHERS- 2020 TEST
Exam Title :
41 ...
Email
Contact

· The architect made sure that the Viceroy’s Palace was higher than Shah Jahan’s Jama Masjid
to assert British dominance/importance.

· New Delhi took nearly 20 years to build.

· Architect Robert Tor Russell designed Connaught Place which was named after the Duke of
Connaught. The construction went on for about 4 years from 1929 to 1933.

(Source: Our Past – III Part – 1, Class VIII NCERT)

QUESTION 35. MjI2MTgwK1BBUlRIK3BhcnRodW1yYWxpeWE1NUBnbWFpbC5jb20rODE0MDA3MTI2OFFVR


NUSU9OIDM0
Who among the following was not a ‘no changer’?

a) Sardar Patel
b) Dr Rajendra Prasad
c) Motilal Nehru
d) Both the option (a) and (b)
Correct Answer: C
Your Answer: Unanswered
Explanation

Solution (c)

Basic Information:

· After Gandhi's arrest (March 1922), there was disintegration, disorganisation and
demoralisation among nationalist ranks.

· A debate started among Congressmen on what to do during the transition period, i.e., the
passive phase of the movement.

· One section led by C.R. Das, Motilal Nehru and Ajmal Khan wanted an end to the boycott of
legislative councils so that the nationalists could enter them to expose the basic weaknesses of
these assemblies and use these councils as an arena of political struggle to arouse popular
enthusiasm.

· Those advocating entry into legislative councils came to be known as the Swarajists. while the
other school of thought led by Vallabhbhai Patel, Rajendra Prasad, C. Rajagopalachari and M.A.
Ansari came to be known as the 'No-changers'.

· The 'No-changers' opposed council entry, advocated, concentration on constructive work, and
continuation of boycott and non-cooperation, and quiet preparation for resumption of the
suspended civil disobedience programme.

· The differences over the question of council entry between the two schools of thought resulted
in the defeat of the Swarajists' proposal of 'ending or mending' the councils at the Gaya session
of the Congress (December 1922).

IASbaba
Score:
Web: http://ilp.iasbaba.com/
41.00 /
Email: ilp@iasbaba.com
Page 38 200
ILP FRESHERS- 2020 TEST
Exam Title :
41 ...
Email
Contact

· C.R Das and Motilal Nehru resigned from the president ship and secretary ship respectively of
the Congress and announced the formation of Congress-Khilafat Swarajya Party, with C.R. Das
as the president and Motilal Nehru as one of the secretaries.

· A compromise was reached at a meeting in Delhi in September 1923. The Swarajists were
allowed to contest elections as a group within the Congress.

(Source: Spectrum)

QUESTION 36. MjI2MTgwK1BBUlRIK3BhcnRodW1yYWxpeWE1NUBnbWFpbC5jb20rODE0MDA3MTI2OFFVR


NUSU9OIDM1
In the Indian history context the terms like ‘chintz’, ‘cossaes’, ‘bandanna’ are related with:

a) Types of printed cotton textiles


b) Types of architectural structures
c) Types of fine silk textiles
d) Types of Indian trade agents helping Europeans.
Correct Answer: A
Your Answer: D
Explanation

Solution (a)

Basic Information:

· Around 1750, before the British conquered Bengal, India was by far the world’s largest
producer of cotton textiles.

· They were extensively traded with Southeast Asia (Java, Sumatra and Penang) and West and
Central Asia.

· The ‘chintz’, ‘cossaes’ (Khassa), ‘bandanna’ were printed cotton cloths.

· Chintz is name derived from the Hindi term Chhint, a cloth with small and colourful flowery
designs. Rich people of England including the Queen herself wore clothes of Indian fabric

· The word bandanna now refers to any brightly coloured and printed scarf for the neck or the
head. Originally, the term derived from the word “Bandhna”.

· Muslin, Calico are other textiles popular with European.

(Source: Our Past – III Part 2, VIII NCERT))

QUESTION 37. MjI2MTgwK1BBUlRIK3BhcnRodW1yYWxpeWE1NUBnbWFpbC5jb20rODE0MDA3MTI2OFFVR


NUSU9OIDM2
Consider the following statements with reference to Gandhi’s view on education in India:

IASbaba
Score:
Web: http://ilp.iasbaba.com/
41.00 /
Email: ilp@iasbaba.com
Page 39 200
ILP FRESHERS- 2020 TEST
Exam Title :
41 ...
Email
Contact

1. He considered the colonial education created a sense of inferiority in the minds of Indians.

2. He strongly felt that Indian languages ought to be the medium of teaching.

3. He emphasised the significance of vocational training in education.

Which of the statements given above is/are correct?

a) 1 and 2 Only
b) 1 and 3 Only
c) 2 and 3 Only
d) 1, 2 and 3
Correct Answer: D
Your Answer: D
Explanation

Solution (d)

Basic Information:

· Gandhi’s ideas on education, called the Basic National Education scheme, have been put
forward in the All India National Education Conference in Wardha on 22nd - 23rd of October,
1937.

· Mahatma Gandhi argued that colonial education created a sense of inferiority in the minds of
Indians. It made them see Western civilisation as superior, and destroyed the pride they had in
their own culture.

· Charmed by the West, appreciating everything that came from the West, Indians educated in
these institutions began admiring British rule.

· Mahatma Gandhi wanted an education that could help Indians recover their sense of dignity
and self-respect.

· Mahatma Gandhi strongly felt that Indian languages ought to be the medium of teaching.

· Western education, Mahatma Gandhi said, focused on reading and writing rather than oral
knowledge; it valued textbooks rather than lived experience and practical knowledge.

· He argued that education ought to develop a person’s mind and soul.

· He claimed that people had to work with their hands, learn a craft, and know how different
things operated. This would develop their mind and their capacity to understand.

(Source: Our Past – III Part 2, VIII NCERT)

QUESTION 38. MjI2MTgwK1BBUlRIK3BhcnRodW1yYWxpeWE1NUBnbWFpbC5jb20rODE0MDA3MTI2OFFVR


NUSU9OIDM3
Consider the following statements about E.V. Ramaswamy Naicker:

IASbaba
Score:
Web: http://ilp.iasbaba.com/
41.00 /
Email: ilp@iasbaba.com
Page 40 200
ILP FRESHERS- 2020 TEST
Exam Title :
41 ...
Email
Contact

1. He was a member of Indian National Congress (INC)

2. He founded the Justice Party.

3. He welcomed Simon Commission and opposed Salt Satyagraha

Which of the statements given above is/are correct?

a) 1 Only
b) 1 and 3 Only
c) 2 Only
d) 2 and 3
Correct Answer: B
Your Answer: B
Explanation

Solution (b)

Basic Information:

· Born in 1879, Periyar is remembered for the Self Respect Movement to redeem the identity
and self-respect of Tamils.

· Periyar started his political career as a Congress worker in his hometown Erode. He
quarrelled with Gandhi over the question of separate dining for Brahmin and non-Brahmin
students at Gurukkulam, a Congress-sponsored school owned by nationalist leader V V S Iyer in
Cheranmahadevi near Tirunelveli.

· After failing to bend the Congress to his view, Periyar resigned from the party in 1925, and
associated himself with the Justice Party and the Self Respect Movement, which opposed the
dominance of Brahmins in social life, especially the bureaucracy.

· Periyar’s fame spread beyond the Tamil region during the Vaikom Satyagraha of 1924. He
would later be referred to as Vaikom Veerar (Hero of Vaikom).

· As a social reformer, he focused on social, cultural and gender inequalities, and his reform
agenda questioned matters of faith, gender and tradition.

· The Self Respect Movement he led promoted weddings without rituals, and sanctioned
property as well as divorce rights for women.

· The Justice Party, officially the South Indian Liberal Federation, was established on November
20, 1916 in Victoria Memorial Hall in Madras by Dr C. Natesa Mudaliar and co-founded by T. M.
Nair and P. Theagaraya Chetty.

Statement Analysis:

Statement 1 Statement 2 Statement 3

Correct Incorrect Correct

IASbaba
Score:
Web: http://ilp.iasbaba.com/
41.00 /
Email: ilp@iasbaba.com
Page 41 200
ILP FRESHERS- 2020 TEST
Exam Title :
41 ...
Email
Contact

He became a member of In the 1940s, Periyar


He staunchly opposed
the Congress, only to leave launched Dravidar
Gandhi. He was among the
it in disgust when he found Kazhagam, which espoused
few leaders who welcomed
that at a feast organised by an independent Dravida
the Simon Commission. He
nationalists, seating Nadu comprising Tamil,
vigorously opposed the Salt
arrangements followed Malayalam, Telugu, and
Satyagraha
caste distinctions Kannada speakers.

(Source: https://www.outlookindia.com/magazine/story/india-news-periyar-ev-ramasamy-the-
man-who-opposed-mahatma-gandhis-idea-of-india/302169 )

QUESTION 39. MjI2MTgwK1BBUlRIK3BhcnRodW1yYWxpeWE1NUBnbWFpbC5jb20rODE0MDA3MTI2OFFVR


NUSU9OIDM4
Which of the following were the demands of Congress in the initial years of its formation?

1. Legislative Councils to be made more representative, given more power, and introduced in
provinces where none existed.

2. Civil service examinations to be held in India as well, not just in London.

3. Separation of the judiciary from the executive

Select the correct code:

a) 1 and 2 Only
b) 1 and 3 Only
c) 2 and 3 Only
d) 1, 2 and 3
Correct Answer: D
Your Answer: Unanswered
Explanation

Solution (d)

Basic Information:

· It has often been said that the Congress in the first twenty years was “moderate” in its
objectives and methods.

· During this period, it demanded a greater voice for Indians in the government and in
administration.

· It demanded that Indians be placed in high positions in the government. For this purpose, it
called for civil service examinations to be held in India as well, not just in London.

· The demand for Indianisation of the administration was part of a movement against racism,
since most important jobs at the time were monopolised by white. Indianisation, it was hoped,
would also reduce the drain of wealth to England.

IASbaba
Score:
Web: http://ilp.iasbaba.com/
41.00 /
Email: ilp@iasbaba.com
Page 42 200
ILP FRESHERS- 2020 TEST
Exam Title :
41 ...
Email
Contact

· The early Congress demanded reduction of revenue, cut in military expenditure, and more
funds for irrigation. It passed many resolutions on the salt tax, treatment of Indian labourers
abroad, and the sufferings of forest dwellers – caused by an interfering forest administration.

· This shows that despite being a body of the educated elite, the Congress did not talk only on
behalf of professional groups, zamindars or industrialists.

QUESTION 40. MjI2MTgwK1BBUlRIK3BhcnRodW1yYWxpeWE1NUBnbWFpbC5jb20rODE0MDA3MTI2OFFVR


NUSU9OIDM5
Consider the following statements about Charles Wood's Education Dispatch, 1854:

1. It recommended the English education policy of ‘downward filtration theory’ for India.

2. Higher education was given a boost in 1857 through the creation of three universities in
Calcutta, Bombay and Madras on the model of the University of London.

3. It laid stress on female and vocational, education, and on teachers' training.

Which of the statements given above is/are correct?

a) 1 and 2 Only
b) 1 and 3 Only
c) 2 and 3 Only
d) 1, 2 and 3
Correct Answer: C
Your Answer: D
Explanation

Solution (c)

Basic Information:

· In 1854, Charles Wood prepared a despatch on an educational system for India. Considered
the "Magna Carta of English Education in. India", this document was the first comprehensive
plan for the spread of education in India.

· It recommended English as the medium of instruction for higher studies and vernaculars at
school level.

· It laid down that the education imparted in government institutions should be secular.

· It recommended a system of grants-in-aid to encourage private enterprise.

· The departments of education were set up in all provinces.

· The ideals and methods of Wood's Despatch dominated the field for five decades which saw
rapid westernisation of education system in India, with educational institutions run by
European headmasters and principals

IASbaba
Score:
Web: http://ilp.iasbaba.com/
41.00 /
Email: ilp@iasbaba.com
Page 43 200
ILP FRESHERS- 2020 TEST
Exam Title :
41 ...
Email
Contact

Statement Analysis:

Statement 1 Statement 2 Statement 3

Incorrect Correct Correct

It systematised the
hierarchy from vernacular
primary schools in villages
It asked the Government of
at bottom, followed by
India to assume
Anglo-Vernacular High It promoted women
responsibility for education
Schools and an affiliated education and even teacher
of the masses, thus
college at the district level, training.
repudiating the 'downward
and affiliating universities
filtration theory'
in the presidency towns of
Calcutta, Bombay and
Madras.

(Source: Spectrum)

QUESTION 41. MjI2MTgwK1BBUlRIK3BhcnRodW1yYWxpeWE1NUBnbWFpbC5jb20rODE0MDA3MTI2OFFVR


NUSU9OIDQw
Which of the following statements is not one of the special features of social and religious
reform movements of the nineteenth century?

a) Movements had remained confined to a narrow social space.


b) Most of the reform movements were headed by and belonged to the three higher castes,
Brahman, Kayastha and Baidya.
c) They relied more on legislation for imposing reform from above.
d) The reformers of the early nineteenth century exhibited a staunch opposition to colonial
rule
Correct Answer: D
Your Answer: B
Explanation

Solution (d)

Basic Information:

· Socio-religious movements of 19 th century like Brahmo Samaj, Young Bengal Movement,


Paramhansa Mandali, Prarthana Samaj, Arya Samaj, etc.

· The reformers of the early nineteenth century exhibited an intrinsic faith in the benevolent
nature of colonial rule and relied more on legislation for imposing reform from above.

IASbaba
Score:
Web: http://ilp.iasbaba.com/
41.00 /
Email: ilp@iasbaba.com
Page 44 200
ILP FRESHERS- 2020 TEST
Exam Title :
41 ...
Email
Contact

· These movements had remained confined to a narrow social space, as the reformist spirit
appealed only to a small elite group, who were primarily the economic and cultural
beneficiaries of colonial rule.

· Socially, they were mostly Hindus, and though caste was not a major criterion for membership,
most of them belonged to the three higher castes, Brahman, Kayastha and Baidya. The Brahmo
movement was almost exclusively patronised by these groups.

· It remained alienated from the masses.

· There was very little or no attempt to create a reformist social consciousness at the grass-
roots level, where religious revivalism later found a fertile ground.

· To summarise their position in Christophe Jaffrelot's words, they "undertook to reform their
society and its religious practices in order to adapt them to Western modernity while preserving
the core of Hindu tradition."

(Source: “From Plassey to Partition: A History of Modern India”)

QUESTION 42. MjI2MTgwK1BBUlRIK3BhcnRodW1yYWxpeWE1NUBnbWFpbC5jb20rODE0MDA3MTI2OFFVR


NUSU9OIDQx
The Kittur uprising (1824) is related with:

a) Rani Chennamma
b) Rani Avantibai
c) Rani Durgawati
d) Rani Rudramma
Correct Answer: A
Your Answer: Unanswered
Explanation

Solution (a)

Basic Information:

· Rani Chennamma, the queen of Kitturu was one such warrior who led a war against British
forces in early part of 19th Century when not many rulers were familiar with the evil designs of
the British.

· Chennamma was born in Kakati, a small village in today’s Belagavi district of Karnataka.

· She became queen of Kitturu (now in Karnataka) when she married Raja Mallasarja of the
Desai family.

· Under the Doctrine of Lapse, British rejected adopted child, Shivalingappa as heir to the
throne.

· The princely state of Kitturu was taken over by the British East India Company in 1824 by
imposing the 'doctrine of lapse', even before it was officially articulated by Lord Dalhousie.

IASbaba
Score:
Web: http://ilp.iasbaba.com/
41.00 /
Email: ilp@iasbaba.com
Page 45 200
ILP FRESHERS- 2020 TEST
Exam Title :
41 ...
Email
Contact

· The British ordered Rani Chennamma to exile the adopted child Shivalingappa, using the
policy of paramountcy and complete authority. But Chennamma defied the order.

· Rani Chennamma fought fiercely with the help of her lieutenants, Sangolli Rayanna and
Gurusiddappa, but was outnumbered and ultimately captured and imprisoned at Bailhongal
Fort, where she died on 21 February 1829.

(Source: https://pib.gov.in/newsite/printrelease.aspx?relid=148944 )

QUESTION 43. MjI2MTgwK1BBUlRIK3BhcnRodW1yYWxpeWE1NUBnbWFpbC5jb20rODE0MDA3MTI2OFFVR


NUSU9OIDQy
Consider the following statements about Deccan Riots in 1875:

1. The peasants specifically targeted bonds, decrees and other related documents dealing with
their debts in the possession of the moneylenders.

2. The peasants imposed social sanctions against those peasants and bullotedars who would not
join the boycott of moneylenders.

3. The Satyasodhak Samaj of Mahatma Jyotirao Phule supported the peasants cause.

Which of the statements given above is/are correct?

a) 1 and 2 Only
b) 1 and 3 Only
c) 2 and 3 Only
d) 1, 2 and 3
Correct Answer: A
Your Answer: Unanswered
Explanation

Solution (a)

Basic Information:

· A major agrarian outbreak occurred in the Poona and Ahmednagar districts of Maharashtra in
1875.

· Like the peasants in other Ryotwari areas, the Deccan peasant also found it difficult to pay
land revenue without getting into the clutches of the moneylender and increasingly losing his
land.

· This led to growing tension between the peasants and the moneylenders most of whom were
outsiders — Marwaris or Gujaratis.

· Three other developments occurred at this time. During the early 1860s, the American Civil
War had led to a rise in cotton exports which had pushed up prices. The end of the Civil War in
1864 brought about an acute depression in cotton exports and a crash in prices.

IASbaba
Score:
Web: http://ilp.iasbaba.com/
41.00 /
Email: ilp@iasbaba.com
Page 46 200
ILP FRESHERS- 2020 TEST
Exam Title :
41 ...
Email
Contact

Simultaneously, in 1867, ‘the Government raised land revenue by nearly 50 per cent. The
situation was worsened by a succession of bad harvests.

· A spontaneous protest movement began in December 1874 in Kardab village in Sirur taluq.

· The peasants organized a complete social boycott of the ‘outsider’ moneylenders to compel
them to accept their demands a peaceful manner.

· The social boycott was soon transformed into agrarian riots when it did not prove very
effective. On 12 May, peasants gathered in Supa, in Bhimthari taluq, on the bazar day and
began a systematic attack on the moneylenders’ houses and shops.

· The colonial government set up Deccan Commission whose recommendations led to passing of
Deccan Agriculturists’ Relief Act of 1879, to extend them a certain protection against the
moneylenders.

Statement Analysis:

Statement 1 Statement 2 Statement 3

Correct Correct Incorrect

The modern nationalist


There was very little intelligentsia of
violence in this settling of Maharashtra supported the
accounts. Once the peasants’ cause. the Poona
Initial focus was on using
moneylenders’ instruments Sarvajanik Sabha, led by
peaceful means like social
of oppression — debt bonds Justice Ranade, had
boycott to avoid violence.
— were surrendered, no organized a successful
need for further violence campaign among the
was felt peasants against the land
revenue settlement of 1867.

(Source: India’s Struggle for Independence)

QUESTION 44. MjI2MTgwK1BBUlRIK3BhcnRodW1yYWxpeWE1NUBnbWFpbC5jb20rODE0MDA3MTI2OFFVR


NUSU9OIDQz
Which of the following leaders were chiefly responsible for the convening of the All India States’
People’s Conference (AISPC)?

1. Baiwantrai Mehta

2. Manikial Kothari

3. G.R. Abhayankar

Select the correct code:

IASbaba
Score:
Web: http://ilp.iasbaba.com/
41.00 /
Email: ilp@iasbaba.com
Page 47 200
ILP FRESHERS- 2020 TEST
Exam Title :
41 ...
Email
Contact

a) 1 Only
b) 2 Only
c) 1 and 2 Only
d) 1, 2 and 3
Correct Answer: D
Your Answer: D
Explanation

Solution (d)

Basic Information:

· Under the influence of the Non-Cooperation and Khilafat Movement launched in 1920
numerous local organizations of the States’ people came into existence.

· Some of the States in which praja mandals or States’ People’s Conferences were organized
were Mysore, Hyderabad, Baroda, the Kathiawad States, the Deccan States, Jamnagar, Indore,
and Nawanagar.

· This process came to a head in December 1927 with the convening of the All India States’
People’s Conference (AISPC) at Bombay which was attended by 700 political workers from the
States. The men chiefly responsible for this initiative were Baiwantrai Mehta, Manikial Kothari
and G.R. Abhayankar.

· The AISPC was a conglomeration of political movements in the princely states of the British
Raj, which were variously called Praja Mandals or Lok Parishads.

· In 1939, Jawaharlal Nehru became president of AISPC, serving in this position till 1946.

(Source: India’s Struggle for Independence)

QUESTION 45. MjI2MTgwK1BBUlRIK3BhcnRodW1yYWxpeWE1NUBnbWFpbC5jb20rODE0MDA3MTI2OFFVR


NUSU9OIDQ0
Consider the following statements about the Federation of Indian Chambers of Commerce and
Industry (FICCI) formed in 1927:

1. The role of the FICCI was seen as that of ‘national guardians of trade, commerce and
industry

2. It generally preferred all forms of struggles – constitutional, mass movements, revolutionary


tactics, etc.

3. It encouraged its members to attend Round Table Conference (RTC)

Which of the statements given above is/are incorrect?

a) 1 and 2 Only
b) 1 and 3 Only
c) 2 and 3 Only

IASbaba
Score:
Web: http://ilp.iasbaba.com/
41.00 /
Email: ilp@iasbaba.com
Page 48 200
ILP FRESHERS- 2020 TEST
Exam Title :
41 ...
Email
Contact

d) 1, 2 and 3
Correct Answer: C
Your Answer: C
Explanation

Solution (c)

Basic Information:

· Since the early 1920s, efforts were being made by various capitalists like G.D. Birla and
Purshottamdas Thakurdas to establish a national level organization of Indian commercial,
industrial and financial interests.

· The FICCI was soon recognized by the British government as well as the Indian public in
general, as representing the dominant opinion as well as the overall consensus within the
Indian capitalist class.

· Indian capitalists had developed a fairly comprehensive economic critique of imperialism in all
its manifestations.

· the Indian capitalist class had its own notions of how the anti-imperialist struggle ought to be
waged. There was the fear that mass civil disobedience, especially if it was prolonged, would
unleash forces which could turn the movement revolutionary in a social sense (i.e., threaten
capitalism itself).

· The capitalists were unwilling to support a prolonged all-out hostility to the government of the
day as it prevented the continuing of day-to-day business and threatened the very existence of
the class.

· They also generally refused to negotiate with the British Government, and certainly to make
any final commitments, on constitutional as well as economic issues, behind the back of the
Congress.

Statement Analysis:

Statement 1 Statement 2 Statement 3

Correct Incorrect Incorrect

In 1930, the FICCI (in


The leaders of the capitalist It was always in favour of
sharp contrast to the
class clearly saw the role of not completely abandoning
Liberals) advised its
the FICCI as being that of the constitutional path and
members to boycott the
‘national guardians of the negotiating table and
Round Table Conference
trade, commerce and generally preferred to put
(RTC) stating that ‘. . .
industry,’ performing in the its weight behind
unless such a conference is
economic sphere in colonial constitutional forms of
attended by Mahatma
India the functions of a struggle as opposed to
Gandhi, as a free man, or
national government. mass civil disobedience.
has at least his approval.”

IASbaba
Score:
Web: http://ilp.iasbaba.com/
41.00 /
Email: ilp@iasbaba.com
Page 49 200
ILP FRESHERS- 2020 TEST
Exam Title :
41 ...
Email
Contact

QUESTION 46. MjI2MTgwK1BBUlRIK3BhcnRodW1yYWxpeWE1NUBnbWFpbC5jb20rODE0MDA3MTI2OFFVR


NUSU9OIDQ1
A group known as ‘responsivists’ split from the Congress-Khilafat Swarajya Party (Swarajists) to
safeguard interests of Hindus. Which of the following leader is not a ‘responsivist’?

a) Lajpat Rai
b) Madan Mohan Malaviya
c) N.C. Kelkar
d) Lokmanya Tilak
Correct Answer: D
Your Answer: Unanswered
Explanation

Solution (d)

Basic Information:

· The Swaraj Party or the Congress-Khilafat Swarajya Party was formed on 1 January 1923 by C
R Das and Motilal Nehru.

· Lokmanya Tilak died on 1 st August, 1920 hence, he was not a part of Swarajya Party.

· By 1924, the Swarajist position had weakened because of widespread communal riots, split
among Swarajists themselves on communal and Responsivist-Non-responsivist lines, and the
death of C.R. Das in 1925 weakened it further.

· The Responsivists among Swarajists—Lala Lajpat Rai, Madan Mohan Malaviya and N.C. Kelkar
—advocated cooperation with the Government and holding of office wherever possible to
protect the socalled Hindu interests. They accused the Non-responsivists like Motilal Nehru of
being anti-Hindu and a beef-eater.

· Thus, non-responsivist of the Swarajya Party reiterated faith in mass civil disobedience and
withdrew from legislatures in March 1926, while another section of Swarajists (responsivists)
went into the 1926 elections as a party in disarray, and did not fare well.

(Source: Spectrum)

QUESTION 47. MjI2MTgwK1BBUlRIK3BhcnRodW1yYWxpeWE1NUBnbWFpbC5jb20rODE0MDA3MTI2OFFVR


NUSU9OIDQ2
Which of the following statement(s) correctly represent the Congress’ objection to the
provisions of Cripps Mission?

1. The provision for Dominion Status rather than full independence.

2. The representation of the princely states in the constituent assembly not by the people of the
states but by the nominees of the rulers.

IASbaba
Score:
Web: http://ilp.iasbaba.com/
41.00 /
Email: ilp@iasbaba.com
Page 50 200
ILP FRESHERS- 2020 TEST
Exam Title :
41 ...
Email
Contact

3. The provision that any province which was not prepared to accept the new constitution would
have the right to sign a separate agreement with Britain

Select the correct code:

a) 1 and 2 Only
b) 1 and 3 Only
c) 3 Only
d) 1, 2 and 3 Only
Correct Answer: D
Your Answer: D
Explanation

Solution (d)

Basic Information:

The Cripps Mission was a failed attempt in late March 1942 by the British government to
secure full Indian cooperation and support for their efforts in World War II. The mission was
headed by a senior minister Sir Stafford Cripps.

Cripps announced that the aim of British policy in India was ‘the earliest possible realization of
self- government in India

But, negotiations between Cripps and the Congress leaders broke down. As Congress objected
the provisions like:

· The Declaration promised India Dominion Status and a constitution-making body after the
War.

· The members to be nominated by the rulers in case of the princely states.

· and above all by the provision for the partition of India.

· The British Government also refused to accept the demand for the immediate transfer of
effective power to the Indians and for a real share in the responsibility for the defence of India.

(Source: India’s Struggle for Independence)

QUESTION 48. MjI2MTgwK1BBUlRIK3BhcnRodW1yYWxpeWE1NUBnbWFpbC5jb20rODE0MDA3MTI2OFFVR


NUSU9OIDQ3
Consider the following statement about INA trials 1945:

1. The defence of the INA prisoners was taken up by Bhulabhaj Desai, Tej Bahadur Sapru, K.N.
Katju, Nehru and Asaf.

2. Nehru considered the captured soldiers of INA as misguided patriots.

3. The Muslim League, Hindu Mahasabha and the Communist Party of India did not support the
INA cause.

IASbaba
Score:
Web: http://ilp.iasbaba.com/
41.00 /
Email: ilp@iasbaba.com
Page 51 200
ILP FRESHERS- 2020 TEST
Exam Title :
41 ...
Email
Contact

Which of the statements given above is/are correct?

a) 1 Only
b) 1 and 2 Only
c) 2 and 3 Only
d) 1, 2 and 3 Only
Correct Answer: B
Your Answer: Unanswered
Explanation

Solution (b)

Basic Information:

· After end of World War 2, the issue which most caught the popular imagination was the fate of
the members of Subhas Chandra Bose’s Indian National Army (INA), who were captured by the
British in the eastern theatre of War.

· The Congress organised an INA Relief and Enquiry Committee, which provided small sums of
money and food to the men on their release, and attempted, though with marginal success, to
secure employment for these men.

· INA Day was observed on 12 November and INA Week from 5 to 11 November 1945.

· Demands for release were raised at kisan Conferences in Dhamangaon and Sholapur on 16
November 1945 and at the tenth session of the All India Women’s Conference in Hyderabad on
29 December 1945.

· Diwali was not celebrated in some areas in sympathy with the INA men.

· Significant sections of Government employees, loyalist sections and even men of the armed
forces were submerged in the tide of pro-INA sentiment.

· Prem Sahgal, Gurubaksh Singh Dhillon and Shah Nawaz Khan - these personalities were
associated with d) Indian National Army trials or Red Fort trials.

Statement Analysis:

Statement 1 Statement 2 Statement 3

Correct Correct Incorrect

IASbaba
Score:
Web: http://ilp.iasbaba.com/
41.00 /
Email: ilp@iasbaba.com
Page 52 200
ILP FRESHERS- 2020 TEST
Exam Title :
41 ...
Email
Contact

The Muslim League, the


The defence of the INA Nehru hailed them as
Communist Party of India,
prisoners was taken up by patriots, albeit misguided,
the Unionist Party, the
the Congress and Akalis, the Justice Party, the
Nehru called for their
Bhulabhaj Desai, Tej Abrars in Rawalpindi, the
judicious treatment by the
Bahadur Sapru, K.N. Katju, Rashtriya Swayamsevak
authorities in view of the
Nehru and Asaf All Sangh, the Hindu
British promise that ‘big
appeared in court at the Mahasabha and the Sikh
changes’ are impending in
historic Red Fort trials. League supported the INA
India.
cause in varying degrees.

(Source: Spectrum)

QUESTION 49. MjI2MTgwK1BBUlRIK3BhcnRodW1yYWxpeWE1NUBnbWFpbC5jb20rODE0MDA3MTI2OFFVR


NUSU9OIDQ4
All India Anti Untouchability League, 1932 was established by:

a) Dr. B.R. Ambedkar


b) M K Gandhi
c) M.C. Rajah
d) Madan Mohan Malaviya
Correct Answer: B
Your Answer: A
Explanation

Solution (b)

Basic Information:

· Gandhi saw the Communal Award as an attack on Indian unity and nationalism.

· Gandhi gave up all his other preoccupations and launched a whirlwind campaign against
untouchability— first from jail and after his release in August 1933 from the outside.

· While in jail, he had set up the All India AntiUntouchability League in September 1932 and
had started the weekly Harijan in January 1933.

· After his release, he shifted to the Satyagraha Ashram in Wardha as he had vowed in 1930 not
to return to Sabarmati Ashram unless swaraj was won.

· Starting from Wardha, he conducted a Harijan tour of the country in the period from
November 1933 to July 1934, covering 20,000 km, collecting money for his newly set up Harijan
Sevak Sangh, and propagating removal of untouchability in all its forms.

· He undertook two fasts— on May 8 and August 16, 1934—to convince his followers of the
seriousness of his effort and the importance of the issue.

(Source: Spectrum)

IASbaba
Score:
Web: http://ilp.iasbaba.com/
41.00 /
Email: ilp@iasbaba.com
Page 53 200
ILP FRESHERS- 2020 TEST
Exam Title :
41 ...
Email
Contact

QUESTION 50. MjI2MTgwK1BBUlRIK3BhcnRodW1yYWxpeWE1NUBnbWFpbC5jb20rODE0MDA3MTI2OFFVR


NUSU9OIDQ5
To rescue British from Congress – Muslim League logjam, Wavell proposed "Breakdown Plan",
its main proposal was:

a) Withdrawal of the British Army and officials to the Muslim provinces of North-West and
North-East and handing over the rest of the country to the Congress.
b) Initiating plebiscite in North-West and North-East, while handing over the rest of the
country to the Congress.
c) Rapid withdrawal of British from India, thus pressurising Congress and Muslim league to
come to a compromise
d) None of the above.
Correct Answer: A
Your Answer: Unanswered
Explanation

Solution (a)

Basic Information:

· The Breakdown Plan was prepared by Lord Wavell and his closest circle of advisors to deal
with the fast evolving political situation in India, which visualised a middle course between
"repression" and "scuttle".

· After the failure of the Shimla Conference in 1945, in pursuance of precisely such a goal, he
came up with a secret scheme which has come to be known in history as Wavell’s ‘Breakdown
Plan’.

· This plan envisaged the withdrawal of the British Army and officials to the Muslim provinces of
North-West and North-East and handing over the rest of the country to the Congress.

· Wavell's Breakdown Plan was formulated with two main goals in mind: Firstly, a safe
withdrawal of the British from India; secondly, to avoid a partition of India by attempting to
maintain it as one geographic entity.

· Although Wavell's overall plan was rejected by the HMG in London, parts of it were, however,
incorporated in the final withdrawal plan laid down by Mountbatten, Wavell's successor, in his
June 3 Plan.

(Source: Spectrum)

QUESTION 51. MjI2MTgwK1BBUlRIK3BhcnRodW1yYWxpeWE1NUBnbWFpbC5jb20rODE0MDA3MTI2OFFVR


NUSU9OIDUw
With respect to ancient Indian Epics, ‘Jaya Samhita’ and ‘Satasahasri Samhita’ are associated
with which famous epic?

a) Upanishads

IASbaba
Score:
Web: http://ilp.iasbaba.com/
41.00 /
Email: ilp@iasbaba.com
Page 54 200
ILP FRESHERS- 2020 TEST
Exam Title :
41 ...
Email
Contact

b) Puranas
c) Ramayana
d) Mahabharata
Correct Answer: D
Your Answer: B
Explanation

Solution (d)

Basic Information:

Epics:

· Epics, or a Mahakavya, are known as the specialty of Sanskrit and are also the earliest forms
of literature. Indian literature is thought to be the earliest literature of the world.

· An epic is supposed to be divided into chapters, or Sarga.

· Every chapter is composed in an individual and specific manner depending on the subject or
theme of the Sarga.

· Epics are thought of as art as much description is provided besides the main plot.

· The two most famous epics are the Mahabharata and the Ramayana.

Mahabharata:

· Mahabharata is older in age and possibly reflects the state of affairs from the 10th century
B.C. to the 4th century A.D.

· Originally it consisted of 8800 verses and was called ‘Jayasamhita’ or the collection of verses
dealing with victory.

· Further, the verses were raised to 24000 and came to be known as Bharata, named after one
of the earliest vedic tribes.

· The final compilation of verses brought the number to one lakh which came to be known as the
‘Mahabharata’ or the ‘Satasahasri Samhita’.

· The final version consists of the narrative, descriptive and didactic material.

· The main narrative is of the Kaurava-Pandava conflict and belongs to later vedic times. The
descriptive portion is used for post vedic times and the didactic portion is for post-Mauryan and
Gupta times.

QUESTION 52. MjI2MTgwK1BBUlRIK3BhcnRodW1yYWxpeWE1NUBnbWFpbC5jb20rODE0MDA3MTI2OFFVR


NUSU9OIDUx
With respect to the post-vedic times literature, which of the following pairs are correctly
matched?

IASbaba
Score:
Web: http://ilp.iasbaba.com/
41.00 /
Email: ilp@iasbaba.com
Page 55 200
ILP FRESHERS- 2020 TEST
Exam Title :
41 ...
Email
Contact

1. Shrautasutras : : Relates to various aspects of measurements for the construction of


sacrificial altars.

2. Grihyasutras : : Relates to the domestic rituals connected with the birth, naming ceremonies,
marriage etc

3. Dharma Sutras : : Relates to life code of conduct.

4. SulvaSutras : : Relates to the big public sacrifices meant for the princes and men of
substance belonging to three higher varnas.

Choose the correct option:

a) 1 and 4
b) 2 and 3
c) 2, 3 and 4
d) 1, 2, 3 and 4
Correct Answer: B
Your Answer: Unanswered
Explanation

Solution (b)

Basic Information:

Post-vedic times have a large corpus of ritual literature.

· Shrautasutras, Grihyasutras and Dharma sutras together form the Kalpasutras.

· Kalpasutras are considered as manuals of Hindu religious practice. A number of them


emerged within the different schools of the Veda, the earliest sacred literature of India.

· Each manual explains the procedures (kalpa) of its school as it applies to three different
categories: the sacrificial ritual-Shrautasutras, the domestic ritual - Grihyasutras and the
conduct of life - Dharmasūtras. They are written in the short aphoristic style of the sutra
(literally “thread”) so that they could be committed easily to memory.

· Kalpa is one of six fields of scholarly discipline known as Vedangas (“accessories to the
Vedas”).

· Both Shrautasutras and Grihyasutras belong to 600-300 BC.

· Sulvasutras prescribe various kinds of measurements for the construction of sacrificial altars.
They mark the beginning of geometry and mathematics.

QUESTION 53. MjI2MTgwK1BBUlRIK3BhcnRodW1yYWxpeWE1NUBnbWFpbC5jb20rODE0MDA3MTI2OFFVR


NUSU9OIDUy
Which of the following can be considered as the source of the Sangam age?

1. Works of Strabo and Pliny

IASbaba
Score:
Web: http://ilp.iasbaba.com/
41.00 /
Email: ilp@iasbaba.com
Page 56 200
ILP FRESHERS- 2020 TEST
Exam Title :
41 ...
Email
Contact

2. The Ashokan Inscriptions

3. Hathigumpha Inscription

4. The Ceylonese Books ‘Mahavamsa’ and ‘Dipavamsa’

Choose the correct option

a) 2 and 3
b) 1 and 4
c) 2, 3 and 4
d) 1, 2, 3 and 4
Correct Answer: D
Your Answer: Unanswered
Explanation

Solution (d)

Basic Information:

· The Sangam Age was the period of history of ancient Tamil Nadu and Kerala and parts of Sri
Lanka (then known as Tamilakam) spanning from c. 6th century BCE to c. 3rd century CE. It
was named after the famous Sangam academies of poets and scholars centered in the city of
Madurai.

· Various literary and archaeological sources form the sangam age sources.

· The Sangam literature chiefly consists of Tholkappiyam, Ettuthogal and Pathuppattu. These
works provide valuable information to know the history of the Sangam Age. Among these
Tholkappiyam was the earliest. During the post-Sangam period, the Pathinen Kilkanakku or the
Eighteen Works was composed. The twin epics - Silappathigaram and Manimegalai - also
belonged to the post- Sangam period. All this literature helps us to know the society, economy
and culture of the ancient Tamils.

· The Asokan Edicts refer to the Chera, Chola and Pandya kingdoms. The Hathikumba
Inscriptions of the Kalinga king , Kharavela also mentions the three Tamil Kingdoms. The
Kalugumalai inscriptions help us to know about ancient Tamil scripts called Tamil Brahms. The
Tirukkovalur inscriptions refer to the local chieftains and the tragic end of the Tamil Poet,
Kapilar. The inscriptions at Thirupparankundrum mention the gift of cave beds to the Jam
monks. The inscriptions found at Arnattar hills, near Pugalur belonged to the First Century A.D.
and these inscriptions furnish information regarding the Chera kings.

· In addition to the Sangam literature, foreign literary accounts remain useful sources for the
study of the Sangam Age. Greek and Roman writers had mentioned the society and economy of
the Sangam Tamils in their accounts. Megasthanes in his book Indica also referred to the three
Tamil Kingdoms. Other authors such as Strabo, Pliny and Ptolemy provide valuable information
regarding the Sangam Age. The Ceylonese books - Mahavamsa and Dipavamsa - help us to fix
the date of the Sangam.

IASbaba
Score:
Web: http://ilp.iasbaba.com/
41.00 /
Email: ilp@iasbaba.com
Page 57 200
ILP FRESHERS- 2020 TEST
Exam Title :
41 ...
Email
Contact

QUESTION 54. MjI2MTgwK1BBUlRIK3BhcnRodW1yYWxpeWE1NUBnbWFpbC5jb20rODE0MDA3MTI2OFFVR


NUSU9OIDUz
Which among the following are correctly matched.

(Author) (Book)

1. Banabhatta Harshacharita

2. Sandhyakara Nandi Ramacharita

3. Bilhana Vikramankadevacharita

4. Kalhana Rajatarangini

Choose the correct option:

a) 1 and 2
b) 2 and 3
c) 1, 2 and 4
d) 1, 2, 3 and 4
Correct Answer: D
Your Answer: Unanswered
Explanation

Solution (d)

Basic Information:

· Banabhatta composed Harshacharita in the seventh century AD. It is a semi-biographical work


describing the early career of Harshavardhana. It gives an excellent idea of the court life of
Harsha and the social and religious life in his age.

· Sandhyakara Nandi wrote Ramacharita. It narrates the story of conflict between the Kaivarta
peasants and the Pala Prince Ramapala.

· Bilhana’s Vikramankadevacharita recounts the achievements of his patron Vikramaditya VI


(1076-1127 AD), the Chalukya king of Kalyan.

· Rajatarangin of “The stream of Kings” was written by Kalhana in the twelfth century. It is a
string of biographies of the kings of Kashmir.

QUESTION 55. MjI2MTgwK1BBUlRIK3BhcnRodW1yYWxpeWE1NUBnbWFpbC5jb20rODE0MDA3MTI2OFFVR


NUSU9OIDU0
Consider the following statements:

1. Chinese travellers both Hsuan Tsang and Fa-Hsien were Buddhists.

2. Fa-Hsien describes the socio-economic conditions of India in the age of Gupta while Hsuan
Tsang presents similar accounts in the age of Harsha.

IASbaba
Score:
Web: http://ilp.iasbaba.com/
41.00 /
Email: ilp@iasbaba.com
Page 58 200
ILP FRESHERS- 2020 TEST
Exam Title :
41 ...
Email
Contact

Which among the above statements is/are correct?

a) 1 only
b) 2 only
c) 1 and 2
d) None
Correct Answer: C
Your Answer: B
Explanation

Solution (c)

Basic Information:

· Both Fa-Hsien and Hsuan Tsang were Buddhists and they came to India to visit the Buddhist
shrines and to study Buddhism.

Fa-Hsien:

· Fa-Hsien was a Chinese Buddhist monk and translator who traveled by foot from Ancient
China to Ancient India, visiting many sacred Buddhist sites in Central Asia, the Indian
subcontinent and Southeast Asia between 399-412 to acquire Buddhist texts.

· Fa-Hsien's visit to India occurred during the reign of Chandragupta II. But he describes
nothing about the ruler or the political condition of his period.

· He is also renowned for his pilgrimage to Lumbini, the birthplace of Gautama Buddha.

· Fa-hien started his voyage to India in 399 A.D. He travelled through the desert of Gobi and
reached Khotan where he found many Buddhist monasteries. He then visited Shanshan. Tarter
Pradesh and Kasagara. The then ruler of Kasagara was a Buddhist. Therefore, he met Buddhist
monks and found many monasteries there also. After that, he crossed the Pamir plateau, Swat
and entered Gandhara Pradesh.

· He reached India about 400 A.D. and remained here up to 411 A.D. He visited peshawar,
Pataliputra, Taxila, Mathura, Kannauj. Sravasti, Kapilavastu, Sarnath and many other places.

· He described the social condition as follows. The people were prosperous and content with
their lives. Public morality was high. Mostly the people were vegetarians and avoided meat and
onions in their meals. They did not use alcohol and other intoxicants. Only Chandalas
(Untouchables), who lived outside cities, engaged in hunting and fishing and were meat eaters.

· He embarked for Ceylon at the sea-port of Tamralipti (West Bengal). He remained in Ceylon
(Sri Lanka) for two years and then reached back China via Jawa in 414 A.D.

Image 1: Fa-hien’s travel route.

IASbaba
Score:
Web: http://ilp.iasbaba.com/
41.00 /
Email: ilp@iasbaba.com
Page 59 200
ILP FRESHERS- 2020 TEST
Exam Title :
41 ...
Email
Contact

Hsuan-Tsang:

· Hsuan-tsang was a Chinese Buddhist monk who in 627 AD traveled overland from China to
India to obtain Buddhist scriptures.

· He returned to China in 643, bringing with him precious manuscripts that he then translated
to Chinese.

· Hsuan-Tsang during his travels, he visited places that we today know as Pakistan, Nepal,
Bangladesh, and India.

· He visited India during the period of emperor Harsha. When he went back to China, he wrote
a detailed description of India during the reign of Harsha in his book ‘Si-yu-ki’ or ‘Record of the
Western Countries’.

· His description has been accepted as the best available source of knowing the administrative,
social and cultural condition of India at that time.

· Hsuan-tsang left detailed accounts of his travels, and also wrote about the interaction between
Chinese Buddhism and Indian Buddhism during the early Tang dynasty. His book “Great Tang
Records on the Western Regions” is a classic in Chinese literature, and contains a lot of first-
hand information about the lands and people found along this stretch of the Silk Road in the 7th
century AD. Roughly 900 years later, the book served as inspiration for the famous novel
“Journey to the West” written by Ming dynasty author Wu Cheng’en.

Statement Analysis:

IASbaba
Score:
Web: http://ilp.iasbaba.com/
41.00 /
Email: ilp@iasbaba.com
Page 60 200
ILP FRESHERS- 2020 TEST
Exam Title :
41 ...
Email
Contact

Statement 1 Statement 2

Correct Correct

Fa-Hsien visited during Chandragupta-II


Both travellers were Buddhists while Hsuan-Tsang visited during Harsha’s
reign.

QUESTION 56. MjI2MTgwK1BBUlRIK3BhcnRodW1yYWxpeWE1NUBnbWFpbC5jb20rODE0MDA3MTI2OFFVR


NUSU9OIDU1
Which among the following are the sites of the Neolithic age in India?

1. Maski

2. Utnur

3. Bramhagiri

4. Belan valley in Mirzapur

5. Harappa and Mohenjadaro

Choose the correct option:

a) 1, 2, 4 and 5
b) 1, 2, 3 and 4
c) 1, 2 and 3
d) 1, 2, 3, 4 and 5
Correct Answer: B
Your Answer: C
Explanation

Solution (b)

Basic Information:

· The term Neolithic comes from two words: neo, or new, and lithic, or stone. As such, this time
period is sometimes referred to as the New Stone Age.

· Though the new stone age began much earlier in 7000 BC but the settlements found in India
are not older than the 6000BC. Some settlements found in South India and eastern India are as
late as 1000BC.

IASbaba
Score:
Web: http://ilp.iasbaba.com/
41.00 /
Email: ilp@iasbaba.com
Page 61 200
ILP FRESHERS- 2020 TEST
Exam Title :
41 ...
Email
Contact

· Based on the types of axes used by the neolithic settlers, three important areas of neolithic
settlements are known.

o Area found in the North in the valley of Kashmir at a place called Burzahom.

o Area found in South India towards south of Godavari.

o Areas in the hills of Assam. (Garo hills)

Additionally neolithic sites have been found in the following areas.

1. On the northern spurs of VIndhyas and Belan valley in Mirzapur and Allahabad districts of
Uttar Pradesh

2. Maski, Brahmagiri, Hallur, Kodekal, Sanganakallu, T.Narsipur and Takkalakota in Karnataka

3. Paiyampalli in Tamilnadu.

4. Piklihal and Utnur in Andhra Pradesh.

Humans in the Neolithic Age still used stone tools and weapons, but they were starting to
enhance their stone tools. There's evidence of initial metallurgy as well, and also creating more
pottery.

But what really distinguishes the New Stone Age from the Old Stone Age, or Paleolithic Age,
that preceded it, is a very important characteristic that is the key feature of the time period: the
invention of agriculture.

QUESTION 57. MjI2MTgwK1BBUlRIK3BhcnRodW1yYWxpeWE1NUBnbWFpbC5jb20rODE0MDA3MTI2OFFVR


NUSU9OIDU2
Consider the following statements with respect to the Chalcolithic phase in India:

1. The Chalcolithic people were acquainted with burnt bricks which were used for constructing
houses.

2. The chalcolithic people used different types of pottery.

3. The chalcolithic people produced wheat and rice along with cotton.

Choose the correct statement/s using the codes given below:

a) 2 and 3
b) 1 only
c) 1, 2 and 3
d) None
Correct Answer: A
Your Answer: Unanswered
Explanation

Solution (a)

IASbaba
Score:
Web: http://ilp.iasbaba.com/
41.00 /
Email: ilp@iasbaba.com
Page 62 200
ILP FRESHERS- 2020 TEST
Exam Title :
41 ...
Email
Contact

Basic Information:

· The end of the neolithic period saw the usage of metals. Copper was the first metal to be used
along with the stone. Such a culture was called Chalcolithic.

· The earliest settlements belonging to this phase are found in South-Eastern Rajasthan, the
western Maharashtra and also in eastern India.

· Ahar and Gilund have been excavated in south eastern Rajasthan.

· Malwa, Kayatha and Eran have been excavated in western Madhya Pradesh.

· Jorwe, Nevasa, Daimabad, Chandoli, Songaon and Inamgaon, Nasik have been excavated in
western Maharashtra.

· People of this period used small tools and weapons made of stone and copper blades. Blade
industry seems to have flourished during this time.

· They used different types of pottery, one of which was called Black and red. It was thrown on a
wheel and occasionally painted white linear designs.

· People domesticated animals and grew food grains like wheat, rice, bajra, lentils including
cotton.

· The people were generally not acquainted with burnt bricks and made their houses with mud.

Statement Analysis:

Statement 1 Statement 2 Statement 3

Incorrect Correct Correct

They made different types of


pottery. People in Madhya
People during this period They produced various food
Pradesh and Maharashtra
built houses of mud. They grains along with cotton,
produced channel-spouted
seldom used burnt bricks. linseed and ber.
pots, dishes on stand and
bowls-on-stand.

QUESTION 58. MjI2MTgwK1BBUlRIK3BhcnRodW1yYWxpeWE1NUBnbWFpbC5jb20rODE0MDA3MTI2OFFVR


NUSU9OIDU3
The religious practices of the people of Indus Valley Civilisation are unique. Which of the
following were probably worshipped by the people of this civilization?

1. Pashupati

IASbaba
Score:
Web: http://ilp.iasbaba.com/
41.00 /
Email: ilp@iasbaba.com
Page 63 200
ILP FRESHERS- 2020 TEST
Exam Title :
41 ...
Email
Contact

2. Mother Goddess

3. Humped Bull

4. Vishnu

5. Pipal tree

Choose the correct option:

a) 1, 2 and 3
b) 1, 2, 3 and 4
c) 1, 2, 3 and 5
d) 1, 2, 3, 4 and 5
Correct Answer: C
Your Answer: C
Explanation

Solution (c)

Basic Information:

· The Indus Valley Civilisation was a Bronze Age civilisation in the north-western regions of
South Asia, lasting from 3300 BCE to 1300 BCE, and in its mature form from 2600 BCE to 1900
BCE.

· It flourished in the basins of the Indus River.

· The civilisation's cities were noted for their urban planning, baked brick houses, elaborate
drainage systems, water supply systems, clusters of large non-residential buildings, and new
techniques in handicraft (carnelian products, seal carving) and metallurgy (copper, bronze,
lead, and tin).

· The large cities of Mohenjo-daro and Harappa very likely grew holded between 30,000 and
60,000 individuals.

· The religious practices of this civilization are unique. Numerous terracotta figurines of women
have been found in the excavated sites. In one figurine a plant is shown growing out of the
embryo of a woman. It probably represents the goddess of earth. Many artefacts relating to
mother-goddess are also found on the sites. The male deity is represented on a seal. This god
has three heads and has horns. He is represented in the sitting posture of a yogi surrounded by
an elephant, tiger, rhino and a buffalo. It is called Pashupati mahadeva. We also come across
prevalence of the phallus worship and female sex organs worship. They also worshipped
animals and trees like peepal trees, humped bull etc.

IASbaba
Score:
Web: http://ilp.iasbaba.com/
41.00 /
Email: ilp@iasbaba.com
Page 64 200
ILP FRESHERS- 2020 TEST
Exam Title :
41 ...
Email
Contact

Image 1: Pashupati Deva

Image 2: Mother Goddess

QUESTION 59. MjI2MTgwK1BBUlRIK3BhcnRodW1yYWxpeWE1NUBnbWFpbC5jb20rODE0MDA3MTI2OFFVR


NUSU9OIDU4
The development of cities was a remarkable feature during the Harappan Civilisation. Which of
the following places can be considered as cities during Harappan Civilisation?

IASbaba
Score:
Web: http://ilp.iasbaba.com/
41.00 /
Email: ilp@iasbaba.com
Page 65 200
ILP FRESHERS- 2020 TEST
Exam Title :
41 ...
Email
Contact

1. Kalibangan

2. Banwali

3. Rangpur

4. Lothal

5. Surkotada

Choose the correct option:

a) 3 and 5
b) 1, 2 and 4
c) 1, 2, 3 and 4
d) 1, 2, 3, 4 and 5
Correct Answer: B
Your Answer: Unanswered
Explanation

Solution (b)

Basic Information:

· Although over 250 harappan sites are known only six can be regarded as cities which includes
Harappa, Mohenjadaro, Chanhudaro, Lothal (Gujrat), Kalibangan (Rajasthan) and Banwali
(Hissar in Haryana).

· Harappa and Mohenjadaro are in Sindh province both forming parts of Pakistan.

· Chanhudaro is about 130 KMS from Mohenjadaro.

· Lothal is in Gujarat at the head of the Gulf of Cambay.

· Kalibangan is in northern Rajasthan.

· Banwali is in Hissar district of Haryana.

QUESTION 60. MjI2MTgwK1BBUlRIK3BhcnRodW1yYWxpeWE1NUBnbWFpbC5jb20rODE0MDA3MTI2OFFVR


NUSU9OIDU5
What does the term ‘Gavisthi’ in Rig Veda refer to?

a) Carpenters
b) Tribal head
c) Unit of Land
d) War
Correct Answer: D
Your Answer: D
Explanation

IASbaba
Score:
Web: http://ilp.iasbaba.com/
41.00 /
Email: ilp@iasbaba.com
Page 66 200
ILP FRESHERS- 2020 TEST
Exam Title :
41 ...
Email
Contact

Solution (d)

Basic Information:

· Rig Veda has many references to cows indicating Aryans being pastoral people.

· Cow was a symbol of wealth and most of the wars used to happen for taking possession of the
cows. Hence the term for war or search for cows in Rig Veda was called ‘Gavisthi’

· Also, any gifts made to the priests were stated in terms of the number of cows.

QUESTION 61. MjI2MTgwK1BBUlRIK3BhcnRodW1yYWxpeWE1NUBnbWFpbC5jb20rODE0MDA3MTI2OFFVR


NUSU9OIDYw
With respect to the Aryans migration into Indian Sub-continent, consider the following
statements:

1. Aryans migrated into Indian Subcontinent in several waves.

2. Aryan tribes maintained harmony amongst themselves which helped them to reign over other
non-Aryan tribes.

Choose the correct statement/s using the options below:

a) 1 only
b) 2 only
c) Both 1 and 2
d) Neither 1 nor 2
Correct Answer: A
Your Answer: C
Explanation

Solution (a)

Basic Information:

· The English word 'Aryan' comes from the Sanskrit word ārya, which is the self-designation
used by the Vedic Indic people who migrated into the Indian subcontinent around 1500 BC.

· Coming from central Asia, this large group of nomadic cattle herders crossed the Hindu Kush
Mountains and came in contact with the Indus Valley Civilization which has reached its peak of
development during that time.

· The Aryans spoke the Indo-European languages.

· On their way to India Aryans first appeared in Iran, where the Indo-Iranians lived for a long
time.

Statement Analysis:

IASbaba
Score:
Web: http://ilp.iasbaba.com/
41.00 /
Email: ilp@iasbaba.com
Page 67 200
ILP FRESHERS- 2020 TEST
Exam Title :
41 ...
Email
Contact

Statement 1 Statement 2

Correct Incorrect

Aryans were involved in two types of


Aryans came to India in several waves. The
conflicts. First, they fought with the pre-
earliest wave is represented by the Rig
Aryans and secondly, they fought amongst
Vedic people who appeared in the sub-
themselves. Intra-tribal conflicts rocked the
continent in 1500 BC. They came into
Aryan communities for a long time. Aryans
conflict with the indegenous inhabitants
were divided into five tribes called
called the ‘dasas’ ‘dasyus’ etc. Dasas seems
panchajana and these tribes fought within
to have been a branch of the early Aryans.
themselves. Sometimes the tribes took the
Probably the ‘Dasyus’ represents the
help of non-aryan tribes to fight with their
original inhabitants.
own tribes.

QUESTION 62. MjI2MTgwK1BBUlRIK3BhcnRodW1yYWxpeWE1NUBnbWFpbC5jb20rODE0MDA3MTI2OFFVR


NUSU9OIDYx
Ashokan inscriptions were written in various scripts. Which of the following were the scripts
used in the Ashokan inscriptions?

1. Brahmi

2. Kharoshti

3. Greek

4. Aramaic

Choose the correct option using code below.

a) 1 and 3
b) 1, 2 and 3
c) 1, 3 and 4
d) 1, 2, 3 and 4
Correct Answer: D
Your Answer: Unanswered
Explanation

Solution (d)

Basic Information:

IASbaba
Score:
Web: http://ilp.iasbaba.com/
41.00 /
Email: ilp@iasbaba.com
Page 68 200
ILP FRESHERS- 2020 TEST
Exam Title :
41 ...
Email
Contact

· The Edicts of Ashoka are a collection of more than thirty inscriptions on the pillars, as well as
boulders and cave walls, attributed to Emperor Ashoka of the Mauryan Empire who reigned
from 268 BCE to 232 BC.

· Three languages were used in the inscriptions. Prakrit, Greek (the language of the
neighbouring Greco-Bactrian kingdom and the Greek communities in Ashoka's realm) and
Aramaic (the official language of the former Achaemenid Empire).

· Four scripts were used. Prakrit inscriptions were written in the Brahmi and Kharosthi scripts,
the latter for the area of modern Pakistan. The Greek and Aramaic inscriptions used their
respective scripts, in the north-western areas of Ashoka's territory, in modern Pakistan and
Afghanistan.

· While most Edicts were in Prakrit, a few were written in Greek or Aramaic.

· The Kandahar Rock Inscription is bilingual Greek-Aramaic. The Kandahar Greek Edict of
Ashoka is in Greek only, and originally probably contained all the Major Rock Edicts 1-14. The
Greek language used in the inscription is of a very high level and displays philosophical
refinement. It also displays an in-depth understanding of the political language of the Hellenic
world in the 3rd century BCE. This suggests the presence of a highly cultured Greek presence
in Kandahar at that time.

· By contrast, in the rock edicts engraved in southern India in the newly conquered territories of
Karnataka and Andhra Pradesh, Ashoka only used the Prakrit of the North as the language of
communication, with the Brahmi script.

QUESTION 63. MjI2MTgwK1BBUlRIK3BhcnRodW1yYWxpeWE1NUBnbWFpbC5jb20rODE0MDA3MTI2OFFVR


NUSU9OIDYy
Which among the following are part of six systems of Hindu philosophy in India?

1. Sankhya

2. Jyotisha

3. Nyaya

4. Vyakarana

5. Yoga

6. Mimamsa

Choose the correct option using the codes given below:

a) 1, 2, 3 and 6
b) 1, 3, 5 and 6
c) 1, 2, 4 and 5
d) 1, 2, 3, 4, 5 and 6
Correct Answer: B

IASbaba
Score:
Web: http://ilp.iasbaba.com/
41.00 /
Email: ilp@iasbaba.com
Page 69 200
ILP FRESHERS- 2020 TEST
Exam Title :
41 ...
Email
Contact

Your Answer: B
Explanation

Solution (b)

Basic Information:

Hindu philosophy refers to philosophies, world views and teachings that emerged in ancient
India. These include six systems –

1. Sankhya : An atheistic and strongly dualist theoretical exposition of consciousness and


matter.

2. Yoga: A school emphasising meditation, contemplation and liberation.

3. Nyaya: Explores sources of knowledge. Nyaya Sutras.

4. Vaisheshika: An empiricist school of atomism. Atomism is a natural philosophy proposing that


the physical world is composed of fundamental indivisible components known as atoms.

5. Mimamsa: Mimaṃsa is a Sanskrit word that means "reflection" or "critical investigation" and
thus refers to a tradition of contemplation which reflected on the meanings of certain Vedic
texts.

6. Vedanta: Literally meaning "end of the Vedas", Vedanta reflects ideas that emerged from the
speculations and philosophies contained in the Upanishads, specifically, knowledge and
liberation.

These are also called the Astika (orthodox) philosophical traditions and are those that accept
the Vedas as an authoritative, important source of knowledge.

Ancient and medieval India was also the source of philosophies that share philosophical
concepts but rejected the Vedas, and these have been called nastika Indian philosophies.
Nastika Indian philosophies include Buddhism, Jainism, Carvaka, Ajivika etc.

Vedangas are six auxiliary disciplines associated with the study and understanding of the Vedas.
They include

1. Shiksha (Phonetics)

2. Kalpa (Ritual Canon)

3. Vyakaran (Grammar)

4. Nirukta (explanation)

5. Chhanda (Vedic meter)

6. Jyotisha (Astrology)

QUESTION 64. MjI2MTgwK1BBUlRIK3BhcnRodW1yYWxpeWE1NUBnbWFpbC5jb20rODE0MDA3MTI2OFFVR


NUSU9OIDYz

IASbaba
Score:
Web: http://ilp.iasbaba.com/
41.00 /
Email: ilp@iasbaba.com
Page 70 200
ILP FRESHERS- 2020 TEST
Exam Title :
41 ...
Email
Contact

The invasion of Iranians during 5th and 6th Century BC brought significant developments in
Northern India. Which of the following developments are related to Iranian Invasion?

1. The invention of the Kharosthi script.

2. It influenced the Mauryan sculpture.

3. It led to Alexander’s invasion of India.

Choose the correct statement/s

a) 1 and 2
b) 1 and 3
c) 2 and 3
d) 1, 2 and 3
Correct Answer: D
Your Answer: Unanswered
Explanation

Solution (d)

Basic information:

· The Achaemenian rulers of Iran taking advantage of the political disunity amongst various
principalities of North-west india like Kambhoja, Gandharas etc invaded India.

· The Iranian ruler Darius penetrated into north-west India in 516 BC and annexed Punjab, west
of Indus and Sindh. This area constituted the 20th province or Satrapy of Iran. The total
number of Satrapys in Iranian empire is 28.

Statement Analysis:

Statement 1 Statement 2 Statement 3

Correct Correct Correct

The Iranian Scribes brought Iranian influence on


into India a form of writing Mauryan sculpture is clearly Through the Iranians, the
which came to be known as discernible. The monument Greeks came to know about
the Kharosthi script. It was of Ashokan times, especially the great wealth of India and
written from right to left. the bell shaped capitals eventually led to the
Some Ashokan inscriptions derived its architecture from Alexanders invasion of India.
are written in this script. Iranian sculpture.

IASbaba
Score:
Web: http://ilp.iasbaba.com/
41.00 /
Email: ilp@iasbaba.com
Page 71 200
ILP FRESHERS- 2020 TEST
Exam Title :
41 ...
Email
Contact

QUESTION 65. MjI2MTgwK1BBUlRIK3BhcnRodW1yYWxpeWE1NUBnbWFpbC5jb20rODE0MDA3MTI2OFFVR


NUSU9OIDY0
With respect to the Northern Black Polished ware (NBP) phase consider the following
statements.

1. NBP phase marked the beginning of second urbanisation in India.

2. It saw the beginning of metallic money for the first time.

Choose the correct statement/s

a) 1 only
b) 2 only
c) Both 1 and 2
d) Neither 1 nor 2
Correct Answer: C
Your Answer: Unanswered
Explanation

Solution (c)

Basic Information:

· The Northern Black Polished Ware culture (abbreviated NBPW or NBP) is an urban Iron Age
Indian culture of the Indian Subcontinent, lasting 700–200 BC, succeeding the Painted Grey
Ware culture and Black and red ware culture.

· It developed beginning around 700 BC, in the late Vedic period, and peaked from 500–300 BC,
coinciding with the emergence of 16 great states or Mahajanapadas in Northern India, and the
subsequent rise of the Mauryan Empire.

· This period is associated with the emergence of Indian subcontinent's first large cities since
the decline of the Indus Valley Civilization. This re-urbanization was accompanied by massive
embankments and fortifications, significant population growth, increased social stratification,
wide-ranging trade networks, specialized craft industries (e.g., carving of ivory, conch shells,
and semi-precious stones), a system of weights, punch-marked coins, and writing in the form of
Brahmi and Kharosthi scripts, including inscribed stamp seals.

· The use of burnt bricks and ringwells appeared in the middle of the NBP phase around 3rd
century BC.

Statement analysis:

Statement 1 Statement 2

Correct Correct

IASbaba
Score:
Web: http://ilp.iasbaba.com/
41.00 /
Email: ilp@iasbaba.com
Page 72 200
ILP FRESHERS- 2020 TEST
Exam Title :
41 ...
Email
Contact

After the Harappan towns disappeared in


1500 BC, for about 1000 years we do not
find any towns in India. With the
appearance of the towns in the middle
gangetic basin in the sixth century BC. Metallic money and punch marked coins
Many towns mentioned in the pali and appeared in trade during this phase.
sanskrit texts such as Kausambi, Sravasti,
Ayodhya, Kapilavastu, Varanasi, Vaisali,
Rajgir, Pataliputra, Champa emerged
during this phase.

QUESTION 66. MjI2MTgwK1BBUlRIK3BhcnRodW1yYWxpeWE1NUBnbWFpbC5jb20rODE0MDA3MTI2OFFVR


NUSU9OIDY1
In vedic times, the terms ‘Nishka’ and ‘Satamana’ refers to

a) Coins
b) Tribes
c) Famous Scholars
d) Women Priests
Correct Answer: A
Your Answer: A
Explanation

Solution (a)

Basic Information:

· Later Vedic periods saw prominent changes in the economic life of people. Life became
sedentary and domestication of animals and cultivation increased.

· Barter system still continued but new coins were introduced in the trade.

· The coins which were in circulation in the latter Vedic era were 'Nishka', 'Satamana', 'Suvarna'
and 'Krishnala'.

· The unit value of goods was a gold bar called “Nishka” weighing three hundred and twenty
ratis, which was also the weight of a Satamana. A ‘Krishnala’ weighed one rati, i.e. 1.8 grams.

QUESTION 67. MjI2MTgwK1BBUlRIK3BhcnRodW1yYWxpeWE1NUBnbWFpbC5jb20rODE0MDA3MTI2OFFVR


NUSU9OIDY2
With regard to the economic system of the Mauryan empire, consider the following statements.

1. Slaves were used in agriculture work.

IASbaba
Score:
Web: http://ilp.iasbaba.com/
41.00 /
Email: ilp@iasbaba.com
Page 73 200
ILP FRESHERS- 2020 TEST
Exam Title :
41 ...
Email
Contact

2. The Mauryas attached greater importance to assessment of taxes than to storage and
depositing.

Choose the correct statement/s

a) 1 only
b) 2 only
c) Both 1 and 2
d) Neither 1 nor 2
Correct Answer: C
Your Answer: C
Explanation

Solution (c)

Basic Information:

· The Mauryas developed a well established economic system.

· Arthashastra of Kautilya mentions that the state appointed 27 superintendents (Adhyakshas)


mostly to regulate the economic activities of the state.

· Each superintendent controlled and regulated specific activities like agriculture, trade and
commerce, weights and measures, weaving and spinning, mining etc.

· The state also helped in developing an elaborate agriculture practice by providing irrigation
facilities and regulated water supply for the benefit of agriculturalists.

· Megasthenes mentions that the officer of the state measured land as in Egypt and inspected
the channels through which water was distributed into smaller channels.

· Trade and commerce during the Mauryan period spread in all directions extending far places
in north-western, southern and north-eastern India.

· They maintained an elaborate taxation system. Many kinds of taxes were collected from
peasants, artisans and traders. The ‘samharta’ was the highest officer in charge of assessment
and ‘Sannidata’ was the chief custodian of the state treasury.

· Epigraphic evidence shows the existence of store houses and that the taxes were collected in
kind and these granaries were meant for helping local people in times of natural calamities like
famine, drought etc.

Statement Analysis:

Statement 1 Statement 2

Correct Correct

IASbaba
Score:
Web: http://ilp.iasbaba.com/
41.00 /
Email: ilp@iasbaba.com
Page 74 200
ILP FRESHERS- 2020 TEST
Exam Title :
41 ...
Email
Contact

According to Arthashastra of Kautilya, a


striking social development of the Maurya
Mauryas attached greater importance to
period was the employment of slaves in
assessment than to storage and depositing
agriculture operations. The state
of taxes. The harm done to the state by the
maintained farms employed slaves for
first is thought to be more serious than the
work. Nearly 150000 war captives brought
harm caused by the second.
by Ashoka from Kalinga may have been
engaged in agriculture

QUESTION 68. MjI2MTgwK1BBUlRIK3BhcnRodW1yYWxpeWE1NUBnbWFpbC5jb20rODE0MDA3MTI2OFFVR


NUSU9OIDY3
With respect to the foreigners who invaded India and settled later after the Mauryan empire,
consider the following statements:

1. The Kushan rulers worshipped Siva, Vishnu and Buddha.

2. ‘The Satrap System’ of Government was introduced by the rulers of Kushans and Sakas.

3. Centralisation of power was the prominent feature of their rule.

Which of the above statement/s is/are correct?

a) 1 and 2
b) 2 and 3
c) 2 only
d) 1, 2 and 3
Correct Answer: A
Your Answer: Unanswered
Explanation

Solution (a)

Basic Information:

Indo-Greeks:

· The first of Central Asian contacts to invade India were the Greeks who were pushed from
their homeland of Bactria by the Scythians. For this reason they are also called Bactrian
Greeks.

· The most famous Indo-Greek ruler was Menander (165-145 BC). He is also known by the name
of Milinda. His capital was at Sakala (modern day Sialkot) in Punjab.

· The monk Nagasena (also known as Nagarjuna) converted Menander to Buddhism.

· The questions posed by Menander to Nagasena and the latter’s replies are recorded in the
form of a book known as Milinda Panha (Pali text) or The Questions of Milinda.

IASbaba
Score:
Web: http://ilp.iasbaba.com/
41.00 /
Email: ilp@iasbaba.com
Page 75 200
ILP FRESHERS- 2020 TEST
Exam Title :
41 ...
Email
Contact

Sakas:

· The Greeks were followed by the Sakas, who controlled a much larger part of India than the
Greeks did. There were five branches of the Sakas spread in Afghanistan and India. These were
in Afghanistan, Punjab (capital at Taxila), Mathura, Western India and fifth branch in Upper
Deccan.

· The most famous Saka ruler was Rudradaman I (130-150 AD). His popularity arises from the
repairs he undertook to improve the Sudarshana lake in the semi-arid zone of Kathiawar.

Parthians:

· The Sakas were followed by the Parthians who came from Iran.

· The Parthian territorial extent was quite small as compared with the Shakas.

· The most famous Parthian ruler was Gondophernes, in whose reign St. Thomas visited India
for the spread of Christianity.

Kushans:

· The Parthians were followed by the Kushans who are also called Yuechis or Tocharians.

· They came from Central Asia.

· Their empire extended from Khorasan in Central Asia to Varanasi in Uttar Pradesh.

· There are two notable Kushan dynasties. The first was the house of Kadphises and who ruled
till AD 50. Their rulers were Kadphises I and Kadphises II. Kadphises I minted copper coins in
imitation of Roman coins.

· The Kushan dynasty followed the Kadphises rulers.

· The most popular Kushan ruler was Kanishka. He started an era in AD 78, which is now known
as the Shaka era and is used by the Government of India. He was a stout follower of Buddhism
and held the fourth Buddhist council in Kashmir in AD 72.

Statement Analysis:

Statement 1 Statement 2 Statement 3

Correct Correct Incorrect

IASbaba
Score:
Web: http://ilp.iasbaba.com/
41.00 /
Email: ilp@iasbaba.com
Page 76 200
ILP FRESHERS- 2020 TEST
Exam Title :
41 ...
Email
Contact

• The kushan rulers


worshipped siva, vishnu and
buddha.
• The sakas and kushans
• Some of the foreign rulers
introduced some curious
were converted to
• The Sakas and Kushans administrative practices like
vaishnavism. Greek
introduced the Satrap two kings ruling in the same
ambassador called
System of government. kingdom.
Heliodorus set up a pillar in
honour of Vishnu near • The empire was divided • Father and Son ruled
Vidisha in Madhya Pradesh. into numerous Satrapies and jointly at one and the same
each satrapy was placed time. Thus it appears that
• The famous greek ruler
under the rule of a satrap. centralisation was not a
Menander was converted to
prominent feature of their
Buddhism.
administration.
• Kushan ruler Vasudeva
was the worshipper of lord
Vishnu.

QUESTION 69. MjI2MTgwK1BBUlRIK3BhcnRodW1yYWxpeWE1NUBnbWFpbC5jb20rODE0MDA3MTI2OFFVR


NUSU9OIDY4
Which of the following terms referring to the administrative system of Satavahanas is/are
correctly matched?

Term Description

1. Amatya : Head of an administrative unit.

2. Ahara : Unit of District.

3. Gaulmika : Head of a military regiment in rural areas.

4. Katavas : Military camps.

Choose the correct option using codes below.

a) 1 and 2
b) 1 and 3
c) 1 only
d) 1, 2, 3 and 4
Correct Answer: D
Your Answer: A
Explanation

Solution (d)

Basic Information:

IASbaba
Score:
Web: http://ilp.iasbaba.com/
41.00 /
Email: ilp@iasbaba.com
Page 77 200
ILP FRESHERS- 2020 TEST
Exam Title :
41 ...
Email
Contact

• Satavahanas followed the Mauryas in Deccan of India.

• Satavahana dynasty ruled from Pune in Maharashtra to Coastal Andhra Pradesh in the second
century BC.

• The earliest inscriptions of the satavahanas belong to the first century BC when they defeated
the Kanvas and established their power in central India.

• The early satavahana kings appeared not in Andhra but in Maharashtra where most of their
inscriptions are found.

• Simuka was the founder of the Satavahana Dynasty and he is believed to have destroyed the
Shunga Power. He did so with the aid of the Rathikas and Bhojakas.

• Gautamiputra satakarni (78-108 AD) was a famous ruler who revived the lost power of
Satavahana and is described as the Destroyer of the Shaka, Pahalava and Yavana Power.

• Satavahanas administration was simple and inspired by the Mauryas. The King was the
protector of the religion and had divine attributes. He possessed the qualities of ancient Gods.

• The Kingdom was divided into the Janapadas and subdivided into Aharas. The ruler of each
Ahara was an Amatya.

• Ahara was divided into Grama which was under the headmen called Gamika.

• Gaulmikas were administrators of the rural areas under the Satavahanas. He headed a
military regiment consisting of nine chariots, nine elephants, 25 horses and 45 foot soldiers.

• Katakas and Skandhavaras were military camps and settlements which served as
administrative centres so long as king was present there.

QUESTION 70. MjI2MTgwK1BBUlRIK3BhcnRodW1yYWxpeWE1NUBnbWFpbC5jb20rODE0MDA3MTI2OFFVR


NUSU9OIDY5
The book ‘Gatasaptasati’ is attributed to which Satavahana King?

a) Nahapana
b) Gautamiputra Satakarni
c) Hala
d) Simuka
Correct Answer: C
Your Answer: Unanswered
Explanation

Solution (c)

Basic Information:

• 'Gatasaptasati' is an ancient collection of Indian poems in the Prakrit language.

IASbaba
Score:
Web: http://ilp.iasbaba.com/
41.00 /
Email: ilp@iasbaba.com
Page 78 200
ILP FRESHERS- 2020 TEST
Exam Title :
41 ...
Email
Contact

• The poems are about love and love's joy. The collection is attributed to the king Hala who
lived in the 1st century.

• The text exists in many versions. Manuscripts have been found in many parts of India in many
languages, far from Maharashtra.

• The poems were changed over time, sometimes deleted and replaced with different poems,
though every manuscript contains exactly 700 poems consistent with the meaning of the title.

QUESTION 71. MjI2MTgwK1BBUlRIK3BhcnRodW1yYWxpeWE1NUBnbWFpbC5jb20rODE0MDA3MTI2OFFVR


NUSU9OIDcw
Consider the following statements with respect to Megalithic phase or Magaliths in India:

1. Megaliths were graves encircled by big pieces of stones.

2. Megalith people practiced an advanced type of agriculture.

3. Few megalithic people buried the skeletons of the dead in Urns made of red pottery in pits.

Which of the above statement/s is/are correct?

a) 1 only
b) 1 and 3
c) 1 and 2
d) 1, 2 and 3
Correct Answer: B
Your Answer: Unanswered
Explanation

Solution (b)

Basic Information:

• Megaliths refer to large stone structures that were constructed either as burial sites or as
commemorative sites.

• In India, archaeologists trace the majority of the megaliths to the Iron Age (1500 BC to 500
BC), though some sites precede the Iron Age, extending up to 2000 BC.

• Megaliths are spread across the Indian subcontinent, though the bulk of them are found in
peninsular India, concentrated in the states of Maharashtra (mainly in Vidarbha), Karnataka,
Tamil Nadu, Kerala, Andhra Pradesh and Telangana.

• The burial sites are the sites with actual burial remains, such as dolmenoid cists (box-shaped
stone burial chambers), cairn circles (stone circles with defined peripheries), and capstones
(distinctive mushroom-shaped burial chambers found mainly in Kerala).

Statement Analysis:

IASbaba
Score:
Web: http://ilp.iasbaba.com/
41.00 /
Email: ilp@iasbaba.com
Page 79 200
ILP FRESHERS- 2020 TEST
Exam Title :
41 ...
Email
Contact

Statement 1 Statement 2 Statement 3

Correct Incorrect Correct

Megalithic phase people


practiced the burying of
goods and artefacts along
Megaliths are big stone with the dead bodies.
structures built either as However, compared to the The megalithic people in the
burial grounds or as number of agricultural tools southern districts of Tamil
commemorative sites. In that were buried, those Nadu buried the skeletons of
India most of them are meant for fighting and the dead in Urns made of
identified with the graves hunting are larger in red pottery in pits.
built of big pieces of stones. number. This shows that
megalithic people did not
practice an advanced type of
agriculture.

Image:

1. Megalithic stone structure in Central India. 2. Urn Burial.

QUESTION 72. MjI2MTgwK1BBUlRIK3BhcnRodW1yYWxpeWE1NUBnbWFpbC5jb20rODE0MDA3MTI2OFFVR


NUSU9OIDcx
With regard to the literature of the Gupta period in India, consider the following statements:

IASbaba
Score:
Web: http://ilp.iasbaba.com/
41.00 /
Email: ilp@iasbaba.com
Page 80 200
ILP FRESHERS- 2020 TEST
Exam Title :
41 ...
Email
Contact

1. All the plays produced during this period were comedies and none of them were tragedies.

2. Characters of the higher and lower classes do not speak the same language.

Which of the above statement/s is/are true?

a) 1 only
b) 2 only
c) Both 1 and 2
d) Neither 1 nor 2
Correct Answer: C
Your Answer: Unanswered
Explanation

Solution (c)

Basic Information:

· The Gupta Empire was an ancient Indian empire existing from the mid-to-late 3rd century AD
to 543 AD.

· At its zenith, from approximately 319 to 543 AD, it covered much of the Indian subcontinent.

· This period is considered as the Golden Age of India by some historians.

· The ruling dynasty of the empire was founded by the king Sri Gupta; the most notable rulers of
the dynasty were Chandragupta I, Samudragupta, and Chandragupta II alias Vikramaditya.

Literature during the period:

· The primary themes of literature during this period were poetry and romantic comedies. To
this period belong the 13 plays written by Bhasa.

· During the Gupta Empire rule, Sanskrit literature was very well received. The Sanskrit
literature was sponsored by many Gupta empire rulers. During his reign, Chandragupta II had
Navaratna, or set of nine poets in his court. The supreme poet among these nine was Kalidasa
who was a great playwright and wrote plays such as AbhignanaShakuntala.

· Religious literature also saw an increase during this period. The two religious epics
Mahabharata and Ramayana were probably compiled around 4th century BC.

· This period also saw the development of Sanskrit grammar based on Panini and Patanjali.
Amarakosa of Amarasimha is one of the prominent works of this period.

Statement Analysis:

Statement 1 Statement 2

IASbaba
Score:
Web: http://ilp.iasbaba.com/
41.00 /
Email: ilp@iasbaba.com
Page 81 200
ILP FRESHERS- 2020 TEST
Exam Title :
41 ...
Email
Contact

Correct Correct

Almost all the plays written during this The language used by various characters
period were comedies. None of them ended depicting various classes in the plays were
in tragedies. different.

QUESTION 73. MjI2MTgwK1BBUlRIK3BhcnRodW1yYWxpeWE1NUBnbWFpbC5jb20rODE0MDA3MTI2OFFVR


NUSU9OIDcy
’Ratnavali’, ‘Nagananda’ and ‘Priyadarsika’ are the literary works of which famous king in
India?

a) Samudragupta
b) Chandragupta II
c) Kanishka
d) Harshavardhana
Correct Answer: D
Your Answer: A
Explanation

Solution (d)

Basic Information:

· Harsha (c. 590–647 AD), also known as Harshavardhana, was an Indian emperor who ruled
North India from 606 to 647 AD.

· He was a member of the Vardhana dynasty.

· His Empire covered much of North and Northwestern India, extended East till Kamarupa, and
South until Narmada River; and eventually made Kannauj (in present Uttar Pradesh state) his
capital, and ruled till 647 AD.

· Harsha was halted by the south Indian Emperor Pulakeshin II of the Chalukya dynasty, when
Harsha tried to expand his Empire into the southern peninsula of India.

· The Chinese traveller Xuanzang (Hsuan-Tsang) visited the court of Harsha and wrote a very
favourable account of him, praising his justice and generosity.

· His biography 'Harshacharita' written by Sanskrit poet Banabhatta, describes his association
with Thanesar, besides mentioning the defence wall, a moat and the palace with a two-storied
Dhavalagriha (white mansion).

· Harsha is widely believed to be the author of three Sanskrit plays Ratnavali, Nagananda and
Priyadarsika.

IASbaba
Score:
Web: http://ilp.iasbaba.com/
41.00 /
Email: ilp@iasbaba.com
Page 82 200
ILP FRESHERS- 2020 TEST
Exam Title :
41 ...
Email
Contact

QUESTION 74. MjI2MTgwK1BBUlRIK3BhcnRodW1yYWxpeWE1NUBnbWFpbC5jb20rODE0MDA3MTI2OFFVR


NUSU9OIDcz
Consider the following statements with respect to the Pandyan Kingdom.

1. Ashokan Inscriptions mentions only about Pandyan Kingdom and there is no mention of Chola
or Chera Kingdoms.

2. The Pandyan kingdom was Matriarchal.

3. Economy of Pandyans shown significant trade in pearls.

Which of the above statement/s is/are correct?

a) 1 and 3
b) 1 and 2
c) 1 only
d) 2 and 3
Correct Answer: D
Your Answer: Unanswered
Explanation

Solution (d)

Basic information:

• The Pandya Dynasty, also known as the Pandyas of Madurai, was a dynasty of south India, one
of the three ethnically Tamil lineages, the other two being the Chola and the Chera.

• The rulers of the three dynasties were referred to as "the three crowned rulers of the Tamil
country.

• The Pandyas ruled extensive territories, at times including the large portions of present-day
south India and Sri Lanka through collateral branches subject to Madurai.

• The Ashokan inscriptions and the sangam literature provide several pieces of evidence of the
Pandyan dynasty as early as 3rd century BC.

Statement Analysis:

Statement 1 Statement 2 Statement 3

Incorrect Correct Correct

IASbaba
Score:
Web: http://ilp.iasbaba.com/
41.00 /
Email: ilp@iasbaba.com
Page 83 200
ILP FRESHERS- 2020 TEST
Exam Title :
41 ...
Email
Contact

Megasthenes speaks of the


pandyan kingdom ruled by a
Megasthenes mentions that
woman and states that seven
The ashokan inscriptions the pandyan kingdom was
year old mothers were found
mentions all three dynasties celebrated for pearls trade.
in the country. Though these
Cholas, Cheras and Pandyas They traded as far as Roman
look exaggerated but
countries.
suggests that the Pandya
society was matriarchal.

QUESTION 75. MjI2MTgwK1BBUlRIK3BhcnRodW1yYWxpeWE1NUBnbWFpbC5jb20rODE0MDA3MTI2OFFVR


NUSU9OIDc0
Consider the following statements with regard to Harsha’s reign in India.

1. Harsha’s administration was more feudal and decentralised than that of the Guptas.

2. Nalanda was the most famous centre of Mahayana Buddhism during Harsha’s reign in India.

Which of the above statement/s is/are correct?

a) 1 only
b) 2 only
c) Both 1 and 2
d) Neither 1 nor 2
Correct Answer: C
Your Answer: Unanswered
Explanation

Solution (c)

Basic Information:

• Harsha (c. 590–647 AD), also known as Harshavardhana, was an Indian emperor who ruled
North India from 606 to 647 AD.

• He was a member of the Vardhana dynasty.

• His Empire covered much of North and Northwestern India, extended East till Kamarupa, and
South until Narmada River; and eventually made Kannauj (in present Uttar Pradesh state) his
capital, and ruled till 647 AD.

• Harsha was halted by the south Indian Emperor Pulakeshin II of the Chalukya dynasty, when
Harsha tried to expand his Empire into the southern peninsula of India.

• The Chinese traveller Xuanzang (Hsuan-Tsang) visited the court of Harsha and wrote a very
favourable account of him, praising his justice and generosity.

Statement Analysis:

IASbaba
Score:
Web: http://ilp.iasbaba.com/
41.00 /
Email: ilp@iasbaba.com
Page 84 200
ILP FRESHERS- 2020 TEST
Exam Title :
41 ...
Email
Contact

Statement 1 Statement 2

Correct Correct

• Harsha united the small republics from


Punjab to central India, and their
representatives crowned him king at an
assembly in April 606 AD, giving him the
title of Maharaja.
• Nalanda was the main centre of learning
• Harsha established an empire that during Harsha's reign.
brought all of northern India under his
• It followed Mahayana Buddhist
control.
philosophy.
• But Harsha continued giving land grants
to priests and officers which made his
administration more feudal and
decentralised.

QUESTION 76. MjI2MTgwK1BBUlRIK3BhcnRodW1yYWxpeWE1NUBnbWFpbC5jb20rODE0MDA3MTI2OFFVR


NUSU9OIDc1
In Jainism, who among the following are called as “Nirgranthis”?

a) The one who lives in temples


b) The one who is free from all bonds
c) Orphans
d) The one who take Sanyasa
Correct Answer: B
Your Answer: B
Explanation

Solution (b)

Basic Information:

In Jainism Nirgranthis is one who is free from all bonds. The followers of he Parsvanath the 23 r
d Thirthankara of Jainism were called as Nirgranthis. Parsvanath was born at Kashi Banaras.
Historically he was separated from Vardhamana by 250 years. Vardhaman himself was born to
Nirgranthi parents (Followers of Parsavnath). The information about his life is received from
Jain texts – Purvas, Angas and Upangas, written in 4th – 6th C AD in Prakrit language

IASbaba
Score:
Web: http://ilp.iasbaba.com/
41.00 /
Email: ilp@iasbaba.com
Page 85 200
ILP FRESHERS- 2020 TEST
Exam Title :
41 ...
Email
Contact

QUESTION 77. MjI2MTgwK1BBUlRIK3BhcnRodW1yYWxpeWE1NUBnbWFpbC5jb20rODE0MDA3MTI2OFFVR


NUSU9OIDc2
Consider the following statements:

1. High phase of universe is called as Utsarpani whereas the low phase of universe is called as
Avsarpani.

2. Soul exists both in living and non – living elements.

Which of the above statements is/are correct as per Jain philosophy?

a) 1 Only
b) 2 Only
c) Both 1 and 2
d) Neither 1 nor 2
Correct Answer: B
Your Answer: C
Explanation

Solution (b)

Basic Information:

Jain Philosophy:

• The Creator: There is no creator. It does not believe in a supreme God. Tirthankars are the
highest authority.

• The Universe: The universe is eternal. It has no beginning and no end and it is moving in a
cyclic fashion. It moves in phases of Highs and Lows:

• Universe is composed of living and non – living elements.

Statement 1 Statement 2

Incorrect Correct

High phase of universe is called as


Avsarpani . D uring this phase people
have a long life (200 – 300 years) and
they are very tall (40-50 ft)
Jain philosophy says that Soul exists both
in living and non – living elements.
whereas the low phase of universe is
called as Utsarpani. During this phase
people have shorter life span (15 – 20
years) and short height (2 -3 feet)

IASbaba
Score:
Web: http://ilp.iasbaba.com/
41.00 /
Email: ilp@iasbaba.com
Page 86 200
ILP FRESHERS- 2020 TEST
Exam Title :
41 ...
Email
Contact

QUESTION 78. MjI2MTgwK1BBUlRIK3BhcnRodW1yYWxpeWE1NUBnbWFpbC5jb20rODE0MDA3MTI2OFFVR


NUSU9OIDc3
Consider the following statements:

1. In Digambara sect male monks do not wear clothes while female monks wear unstitched
plain white sarees.

2. In Svetambaras follow only 4 vows out of 5 vows, they do not follow “aparigraha”

Which of the above statements is/are correct?

a) 1 Only
b) 2 Only
c) Both 1 and 2
d) Neither 1 nor 2
Correct Answer: A
Your Answer: Unanswered
Explanation

Solution (a)

Basic Information:

Jain order has been divided into two major sects: Digambara and Svetambara. The division
occurred mainly due to famine in Magadha which compelled a group led by Bhadrabahu to
move South India. During the 12 years famine, the group in South India stick to the strict
practices while the group in Magadha adopted a more lax attitude and started wearing white
clothes. After the end of famine, when the Southern group came back to Magadha, the changed
practices led to the division of Jainism into two sects.

Digambara Svetambara

· Follow all five vows (Satya, Ahimsa,


Asteya, Aparigraha and Brahmacharya).
· Monks wear white clothes.
· Believe women cannot achieve
· Believe women can achieve liberation.
liberation.
· Sthulabhadra was an exponent of this
· Bhadrabahu was an exponent of this
sect.
sect.
Major Sub-Sects
Major Sub-Sects
Murtipujaka, Sthanakvasi, Terapanthi
Mula Sangh, Bisapantha, Terapantha,
Taranpantha or Samaiyapantha

IASbaba
Score:
Web: http://ilp.iasbaba.com/
41.00 /
Email: ilp@iasbaba.com
Page 87 200
ILP FRESHERS- 2020 TEST
Exam Title :
41 ...
Email
Contact

Statement 1 Statement 2

Correct Incorrect

Monks of this sect believe in complete


In Svetambaras follow only 4 vows out of
nudity. Male monks do not wear clothes
5 vows they do not follow
while female monks wear unstitched plain
“brahmacharya”
white sarees.

QUESTION 79. MjI2MTgwK1BBUlRIK3BhcnRodW1yYWxpeWE1NUBnbWFpbC5jb20rODE0MDA3MTI2OFFVR


NUSU9OIDc4
Consider the following pairs:

Mudras of Buddha Term represent

1. Varada Mudra Fulfilment of all wishes, the gesture of charity

2. Abhaya Mudra Reassuaranc, blessing and protection

3. Vitarka Mudra Intallectual arguments, discussion.

Which of the above pairs is/are correctly matched?

a) 2 only
b) 1 and 2 only
c) 2 and 3 only
d) 1, 2 and 3
Correct Answer: D
Your Answer: D
Explanation

Solution (d)

Basic Information:

IASbaba
Score:
Web: http://ilp.iasbaba.com/
41.00 /
Email: ilp@iasbaba.com
Page 88 200
ILP FRESHERS- 2020 TEST
Exam Title :
41 ...
Email
Contact

QUESTION 80. MjI2MTgwK1BBUlRIK3BhcnRodW1yYWxpeWE1NUBnbWFpbC5jb20rODE0MDA3MTI2OFFVR


NUSU9OIDc5
Consider the following statements:

1. According to Buddhism, there are eight noble truths (Arya Satya)

2. 4th Buddhist Council held at Pataliputra presided by Vasumitra and Ashwaghosha

Which of the above statements is/are correct?

a) 1 Only
b) 2 Only
c) Both 1 and 2
d) Neither 1 nor 2
Correct Answer: D
Your Answer: A
Explanation

Solution (d)

Basic Information:

IASbaba
Score:
Web: http://ilp.iasbaba.com/
41.00 /
Email: ilp@iasbaba.com
Page 89 200
ILP FRESHERS- 2020 TEST
Exam Title :
41 ...
Email
Contact

· 1st Council: Period: 483 BC- just after the death of Buddha. Place – Rajgriha. Presided by
Mahakasappa

· 2nd Council: Period: 383 BC, Place: Vaishali, Presided by Shatakhambri

· 3rd Council: Period: 250-262 BC, Place: Patliputra, Presided by Moghiputta Tissa

· 4th Council: Period: 1st Century AD, Place: Kunzalwar, Presided by Vasumitra and
Ashwaghosha. At the fourth council the Buddhism got divided into two sects: Hinayana and
Mahayana

Statement 1 Statement 2

Incorrect Incorrect

According to Buddhism, there are four


noble truths (Arya Satya)

· Existence of Sorrow – Dukkha

· Cause of Sorrow – Desire or Maya 4th Council: Place: Kunzalwar

· There is a way to get rid of Sorrow – Presided by Vasumitra and Ashwaghosha


Hope

· Follow the eight-fold path to get Nirvana


and Mahaparinirvana

QUESTION 81. MjI2MTgwK1BBUlRIK3BhcnRodW1yYWxpeWE1NUBnbWFpbC5jb20rODE0MDA3MTI2OFFVR


NUSU9OIDgw
Consider the following statements:

1. White spotted Red sandstone are used in Gandhara School of arts whereas white marble is
used in Amaravati School of arts

2. Buddha’s images of showing Abhaymudra and Dharmachakraparivartana Mudra can be seen


in Mathura school of arts

Which of the above statements is/are incorrect ?

a) 1 Only
b) 2 Only
c) Both 1 and 2
d) Neither 1 nor 2
Correct Answer: A
Your Answer: C

IASbaba
Score:
Web: http://ilp.iasbaba.com/
41.00 /
Email: ilp@iasbaba.com
Page 90 200
ILP FRESHERS- 2020 TEST
Exam Title :
41 ...
Email
Contact

Explanation

Solution (a)

Basic Information:

Features Mathura Gandhara Amravati

Time period 2nd C. BC 1st C. BC 1st C. BC

North western part of Valley of Krishna


Flourished in Mathura region
Indian subcontinent and Godavari

Initially local rulers Vakatakas and


Patronization
but later by Shakas Shakas and Kushanas
by
and Kushanas Ikshavakus

Greeko-Roman and so
Initially indigenous
but after the Shakas also known as Greeko No one;
and Kushanas
Influenced by Roman, Indo-Roman Indigenous from
patronization,
influence of and Greeko Buddhist beginning to end.
GreekoRoman
school of art.

Type of Stone White spotted Red


Blue-Grey Stone White marble
used sandstone

Facial expression
Physical beauty Physical beauty
· Clean shave and
· Muscular body · Expression
Emphasis or head
· Transparent · through
focus on · Abhaymudra and
· cloth · postures
Dharmachakraparivar
· Curly hairs · Erotic images
tana Mudra

Statement 1 Statement 2

IASbaba
Score:
Web: http://ilp.iasbaba.com/
41.00 /
Email: ilp@iasbaba.com
Page 91 200
ILP FRESHERS- 2020 TEST
Exam Title :
41 ...
Email
Contact

Incorrect Correct

White spotted Red sandstone are used in Buddha’s images of showing Abhaymudra
Mathura school of arts whereas white and Dharmachakraparivartana Mudra can
marble is used in Amaravati school of arts be seen in Mathura school of arts

QUESTION 82. MjI2MTgwK1BBUlRIK3BhcnRodW1yYWxpeWE1NUBnbWFpbC5jb20rODE0MDA3MTI2OFFVR


NUSU9OIDgx
Consider the following places:

1. Kalinga

2. Kakinada

3. Suraprakara

4. Girinagara

In which of the above places pillar edicts were built by Ashoka?

a) 1 and 2 Only
b) 2 and 3 Only
c) 1, 2, and 3 only
d) 1, 2, 3 and 4
Correct Answer: A
Your Answer: Unanswered
Explanation

Solution (a)

Basic Information:

IASbaba
Score:
Web: http://ilp.iasbaba.com/
41.00 /
Email: ilp@iasbaba.com
Page 92 200
ILP FRESHERS- 2020 TEST
Exam Title :
41 ...
Email
Contact

QUESTION 83. MjI2MTgwK1BBUlRIK3BhcnRodW1yYWxpeWE1NUBnbWFpbC5jb20rODE0MDA3MTI2OFFVR


NUSU9OIDgy
Consider the following statements:

1. Vardhaman attained the Supreme knowledge (Kaivalya) under a Sal tree in

2. 498 BC at a place called Trimbhigram on the bank of River Rijupalika

3. Vardhaman mahavir’s son-in-law was his last disciple

Which of the above statements is/are correct?

a) 1 Only
b) 2 Only

IASbaba
Score:
Web: http://ilp.iasbaba.com/
41.00 /
Email: ilp@iasbaba.com
Page 93 200
ILP FRESHERS- 2020 TEST
Exam Title :
41 ...
Email
Contact

c) Both 1 and 2
d) Neither 1 nor 2
Correct Answer: A
Your Answer: Unanswered
Explanation

Solution (a)

Basic Information:

· Vardhaman mahavira was a contemporary of Gautama Buddha.

· Initially he lived life of household but at the age of 30 he abandoned house in search of the
truth and became an ascetic. For the next 12 years he meditated and practiced the austerities
of life.

· At the age of 42 years, he attained the Supreme knowledge (Kaivalya) under a Sal tree in

· 498 BC at a place called Trimbhigram on the bank of River Rijupalika (Jharkhand). Through
kaivalya he conquered misery and happiness. Because of this conquest he is known as Mahavira
or the great hero or jina i.e. the conqueror of senses, and his followers are known as Jainas.

Statement 1 Statement 2

Correct Incorrect

Vardhaman attained the Supreme


knowledge (Kaivalya) under a Sal tree in Vardhaman mahavir’s son in law was his
first disciple. He married to vardhaman’s
498 BC at a place called Trimbhigram on daughter Priyadharshika
the bank of River Rijupalika.

QUESTION 84. MjI2MTgwK1BBUlRIK3BhcnRodW1yYWxpeWE1NUBnbWFpbC5jb20rODE0MDA3MTI2OFFVR


NUSU9OIDgz
Consider the following statements:

1. Not to see the opposite sex.

2. Not to talk to the opposite sex.

3. Not to touch the opposite sex.

4. Not to feel about the opposite sex.

Which of the above statements are not followed in Jainism as per fifth doctrines?

a) 1 and 2 Only

IASbaba
Score:
Web: http://ilp.iasbaba.com/
41.00 /
Email: ilp@iasbaba.com
Page 94 200
ILP FRESHERS- 2020 TEST
Exam Title :
41 ...
Email
Contact

b) 2and 3 Only
c) 1, 2, 3 and 4
d) None
Correct Answer: D
Your Answer: D
Explanation

Solution (d)

Basic Information:

Five doctrines of Jainism –

1. Non – Violence

2. Not to Steal

3. Not to Accumulate

4. Not to tell a lie

5. Celibacy: Celibacy in Jainism is slightly different than that of Hinduism. In

Hinduism celibacy means not to marry or not to have a relationship with the

other sex.

a. In Jainism, apart from the above two, some more conditions are there like:

1. Not to see the opposite sex.

2. Not to talk to the opposite sex.

3. Not to touch the opposite sex.

4. Not to feel about the opposite sex.

Only 5th doctrine was added by Mahavira, the other 4 were taken over by him form

previous teachers.

QUESTION 85. MjI2MTgwK1BBUlRIK3BhcnRodW1yYWxpeWE1NUBnbWFpbC5jb20rODE0MDA3MTI2OFFVR


NUSU9OIDg0
Consider the following:

1. Right knowledge

2. Right attain moksha

3. Right action

4. Right faith

IASbaba
Score:
Web: http://ilp.iasbaba.com/
41.00 /
Email: ilp@iasbaba.com
Page 95 200
ILP FRESHERS- 2020 TEST
Exam Title :
41 ...
Email
Contact

Which of the above statements are considered as triratnas of Jainism?

a) 1, 2 and 3 Only
b) 1, 3 and 4 Only
c) 2, 3 and 4
d) None
Correct Answer: B
Your Answer: B
Explanation

Solution (b)

Basic Information:

Main aim of Jainism is to get freedom from worldly bonds. This can be achieved by 3 jewels or
triratna of Jainism and no rituals are required.

3 jewels or triratnas of Jainism –

• Right knowledge

• Right faith

• Right action or conduct (5 doctrines are part of it)

QUESTION 86. MjI2MTgwK1BBUlRIK3BhcnRodW1yYWxpeWE1NUBnbWFpbC5jb20rODE0MDA3MTI2OFFVR


NUSU9OIDg1
Consider the following pairs:

Phases of Buddha’s life Term represents

1. Mahabhinishkramana Enlightenment

2. Nirvana The act of leaving the house

3. Dharma Chakra Parivartan Setting the wheel in motion

IASbaba
Score:
Web: http://ilp.iasbaba.com/
41.00 /
Email: ilp@iasbaba.com
Page 96 200
ILP FRESHERS- 2020 TEST
Exam Title :
41 ...
Email
Contact

Which of the above pairs is/are correctly matched?

a) 3 only
b) 1 and 3 only
c) 2 and 3 only
d) 1, 2 and 3
Correct Answer: A
Your Answer: D
Explanation

Solution (a)

Basic Information:

Four sites of Buddha:

Old man, sick man, dead body, a monk

These sites made him realize that there are sorrows in the world. And he wanted to find the

solution for these.

Pair 1 Pair 2 Pair 3

Incorrect Incorrect Correct

Siddhartha got
At 29, Buddha left his enlightenment and became
house with a charioteer ‘Buddha’ that is the
‘ Dharma Chakra
‘Channa’ and a horse enlightenment. This is
Parivartan’ it means setting
‘Kanthaka’. The act of called the state of Nirvana
the wheal in motion.
leaving the house is called In Buddhism (Note: it is
Mahabhinishkramana’. different from the Nirvana
of Jainism)

QUESTION 87. MjI2MTgwK1BBUlRIK3BhcnRodW1yYWxpeWE1NUBnbWFpbC5jb20rODE0MDA3MTI2OFFVR


NUSU9OIDg2
Which of the following literary works of Vishakadatta that narrates the ascent of the king
Chandragupta Maurya?

a) Vikramorvasiyam
b) Kumarasambhavam
c) Rithusamharam
d) Mudrarakshasam
Correct Answer: D

IASbaba
Score:
Web: http://ilp.iasbaba.com/
41.00 /
Email: ilp@iasbaba.com
Page 97 200
ILP FRESHERS- 2020 TEST
Exam Title :
41 ...
Email
Contact

Your Answer: D
Explanation

Solution (d)

Basic Information:

Vikramorvasiya Kumarasambhava
Rithusamharam Mudrarakshasam
m m

It was written by
It talks about the The poem has six
It talks about the Vishakadatta. It
birth of Kumara canton for the six
vedic love story of narrates the ascent of
(Karthikeya), the Indian seasons: grism,
king Pururavas the king Chandragupta
son of lord Shiva varsa, sarat, hemanta,
and an Apsara? Mourya to power in
and Parvathi vasanta
India

QUESTION 88. MjI2MTgwK1BBUlRIK3BhcnRodW1yYWxpeWE1NUBnbWFpbC5jb20rODE0MDA3MTI2OFFVR


NUSU9OIDg3
The Travellers who visited India in between the tenth and thirteenth centuries A. D.

a) Al- Beruni, Huin-Tsang, and Meghasthanise


b) Fa-Hien, Huin-Tsang, and Meghasthanise
c) Al- Beruni, Al-Masudi, and Marco Polo
d) Fa-Hien, Al- Beruni, and Al- Beruni
Correct Answer: C
Your Answer: Unanswered
Explanation

Solution (c)

Basic Information:

Fa-Hien visited India during 405-411 A.D.

Huin-Tsang visited India during 630-645 A.D.

Meghasthanise visited India during 302-298 B.C.

Al- Beruni Al-Masudi Marco Polo

IASbaba
Score:
Web: http://ilp.iasbaba.com/
41.00 /
Email: ilp@iasbaba.com
Page 98 200
ILP FRESHERS- 2020 TEST
Exam Title :
41 ...
Email
Contact

Al- Beruni or Abu Rehan


Mahamud. Visited India
during 1024-1030 A.D.
Visited India during 957 A.D. Visited India during 1292-129
He was a Muslim Scholar He was an Arab traveller. 4 A.D. He was a Venetian
and Polymath. First Traveller. His work “The Book
Gives an extensive account of of Sir Marco Polo” which
Muslim Scholar to study
India in his work “Muruj-ul- gives an invaluable account of
India also known as
Zahab”. the economic history of India.
founder of Indology.
Wrote ”Tahqiq-i-Hind/
Kitab-ul-Hind.

QUESTION 89. MjI2MTgwK1BBUlRIK3BhcnRodW1yYWxpeWE1NUBnbWFpbC5jb20rODE0MDA3MTI2OFFVR


NUSU9OIDg4
Consider the following statements:

1. Ramananda was first Bhakti saint and founder of Bhakti Movement of northern India.

2. He was a disciple of Shankaracharya

Which of the above statements is/are incorrect ?

a) 1 Only
b) 2 Only
c) Both 1 and 2
d) Neither 1 nor 2
Correct Answer: B
Your Answer: Unanswered
Explanation

Solution (b)

Basic Information:

Ramananda was a 14 th -century Vaishnava devotional poet saint, in the Ganges river region of
northern India. The Hindu tradition recognizes him as the founder of the Ramanandi
Sampradaya, the largest monastic Hidnu renunciant community in modern times

Statement 1 Statement 2

Correct Incorrect

IASbaba
Score:
Web: http://ilp.iasbaba.com/
41.00 /
Email: ilp@iasbaba.com
Page 99 200
ILP FRESHERS- 2020 TEST
Exam Title :
41 ...
Email
Contact

Ramananda was first Bhakti saint and


founder of Bhakti Movement of northern He was a disciple of Ramanuja
India.

QUESTION 90. MjI2MTgwK1BBUlRIK3BhcnRodW1yYWxpeWE1NUBnbWFpbC5jb20rODE0MDA3MTI2OFFVR


NUSU9OIDg5
Consider the following pairs:

Author Work

1. Vishnu Sharma Panchasiddhantika

2. Vrahamihir Panchatantra

3. Vatsyana Kamasutra

Which of the above pairs is/are correctly matched?

a) 1 and 3 only
b) 2 and 3 only
c) 1, 2 and 3
d) 3 only
Correct Answer: D
Your Answer: D
Explanation

Solution (d)

Basic Information:

Pair 1 Pair 2 Pair 3

Incorrect Incorrect Correct

Vishnu Sharma wrote Vatsyana wrote Kamasutra.


Vrahamihir wrote
Panchatantrait contains Book on 64 different arts and
Panchasiddhantika this book
They short animal stories crafts it includes – music,
deals with astronomy and
to build social values and sculptor, drama and
astrology.
morals. lovemaking

QUESTION 91. MjI2MTgwK1BBUlRIK3BhcnRodW1yYWxpeWE1NUBnbWFpbC5jb20rODE0MDA3MTI2OFFVR


NUSU9OIDkw
Consider the following statements:

IASbaba
Score:
Web: http://ilp.iasbaba.com/
41.00 /
Email: ilp@iasbaba.com
Page 100 200
ILP FRESHERS- 2020 TEST
Exam Title :
41 ...
Email
Contact

1. Bashara is one of the order of Sufism which means those who obeyed the Islamic laws.

2. Suhrwardi is one of the twelve Silsila under Beshara

3. In Sufi term, Urs means follower

Which of the above statements is/are correct?

a) 1 and 3 only
b) 2 and 3 only
c) 1 only
d) 3 only
Correct Answer: C
Your Answer: Unanswered
Explanation

Solution (c)

Basic Information:

Sufism is a mystical form of Islam, a school of practice that focuses on the spiritual search for
God and shuns materialism. It is a form of Islamic mysticism which stresses on asceticism.
There is a lot of emphasis on love and devotion towards God.

Statement 1 Statement 2 Statement 3

Correct Incorrect Incorrect

Bashara is one of the Suhrwardi is one of the In sufi term, Urs means death
order of Sufism which twelve silsila under Bashara.
means those who obeyed Chisti is also one of the most Whereas Sama means
the Islamic laws. important silsila of sufism Musical gathering

QUESTION 92. MjI2MTgwK1BBUlRIK3BhcnRodW1yYWxpeWE1NUBnbWFpbC5jb20rODE0MDA3MTI2OFFVR


NUSU9OIDkx
Consider the following statements:

1. Tripartite Struggle is related to the Palasin the east, the Rashtrakutas in the Deccan, the
Delhi sulthanates in the northern most region

2. Vikramshila University was founded by Dharmpala.

Which of the above statements is/are correct?

a) 1 Only
b) 2 Only

IASbaba
Score:
Web: http://ilp.iasbaba.com/
41.00 /
Email: ilp@iasbaba.com
Page 101 200
ILP FRESHERS- 2020 TEST
Exam Title :
41 ...
Email
Contact

c) Both 1 and 2
d) Neither 1 nor 2
Correct Answer: B
Your Answer: Unanswered
Explanation

Solution (b)

Basic Information:

After Harshavardhana, the Rajputs emerged as a powerful force in Northern India. The
important Rajput kingdoms are Pala dynasty founded by Gopala with capital city as Muddagiri,
Rashtrkuta dynasty founded by Dantidurga with capital city as Manyaketh, Pratihara dynasty
founded by Nagabhatta- I with Avanti as a capital city.

Statement 1 Statement 2

Incorrect Correct

The Tripartite Struggle for control of


Vikramshila University was founded by
northern India took place in the ninth
Dharmpala. Dharmapala was h eson of
century. The struggle was between the
Gopala who was the founder of pala
Pratihara Empire in the North, the Pala
dynasty. He also revived Nalanda
Empire in the west and the Rashtrakuta
unoversity
Empire in the Deccan

QUESTION 93. MjI2MTgwK1BBUlRIK3BhcnRodW1yYWxpeWE1NUBnbWFpbC5jb20rODE0MDA3MTI2OFFVR


NUSU9OIDky
which of the following book gives an account of the Tughlaq dynasty?

a) Tughlaq-Nama
b) Fatawa-i-jahandari
c) Kitab-fi-Tahqiq
d) Qamas
Correct Answer: B
Your Answer: A
Explanation

Solution (b)

Basic Information:

• Tughlaq-Nama: Written by Amir Khusrau gives an account of Ghiyasuddin’s reign

• Fatawa-i-jahandari: Written by Ziauddin barani gives an account of the Tughlaq dynasty

IASbaba
Score:
Web: http://ilp.iasbaba.com/
41.00 /
Email: ilp@iasbaba.com
Page 102 200
ILP FRESHERS- 2020 TEST
Exam Title :
41 ...
Email
Contact

• Kitab-fi-Tahqiq: Written by Alberuni this book is about the Indian Science

• Qamas: Written by Firozabadi it is a Arabic words dictionary

QUESTION 94. MjI2MTgwK1BBUlRIK3BhcnRodW1yYWxpeWE1NUBnbWFpbC5jb20rODE0MDA3MTI2OFFVR


NUSU9OIDkz
Malik Ahmad was the founder of which of the following kingdoms?

a) Imad Shahi Dynasty


b) Adil Shahi Dynasty
c) Nizam Shahi Dynasty
d) Barid Shahi Dynasty
Correct Answer: C
Your Answer: Unanswered
Explanation

Solution (c)

Basic Information:

• Imad Shahi dynasty was founded by Fataullah imad Shahi

• Adil Shahi dynasty was founded by Yusuf Adil Shahi

• Nizam Shahi dyanasty was founded by Malik Ahmad

• Barid Shahi dynasty was founded by Amir Ali Barid

• Qutub Shahi dynasity was founded by Quli Qutub Shah

QUESTION 95. MjI2MTgwK1BBUlRIK3BhcnRodW1yYWxpeWE1NUBnbWFpbC5jb20rODE0MDA3MTI2OFFVR


NUSU9OIDk0
Consider the following:

a) Slave Dynasty
b) Sayyid Dynasty
c) Tughluq Dynasty
d) Lodhi Dynasty
e) Khilji Dynasty
Which of the following is the correct chronological order of the Delhi Sulthanates?
f) 1, 2,3 4 and 5
g) 1, 5, 3, 2 and 4
h) 1, 2, 5, 3 and 4

IASbaba
Score:
Web: http://ilp.iasbaba.com/
41.00 /
Email: ilp@iasbaba.com
Page 103 200
ILP FRESHERS- 2020 TEST
Exam Title :
41 ...
Email
Contact

i) 1, 3, 2, 5 and 4
Correct Answer: B
Your Answer: F
Explanation

Solution (b)

Basic Information:

• Slave Dynasty was founded in 1206 by Qutubuddin Aibak

• Khilji Dynasty was founded in 1290 by Jalaluddin Khilji

• Tughluq Dynasty was founded in 1320 by Ghiyasuddin Tughkuq

• Sayyid Dynasty was founded in 1414 by Khizr Khan

• Lodhi Dynasty was founded in 1451 by Bahlol Lodhi

QUESTION 96. MjI2MTgwK1BBUlRIK3BhcnRodW1yYWxpeWE1NUBnbWFpbC5jb20rODE0MDA3MTI2OFFVR


NUSU9OIDk1
Consider the following statements:

1. The blue water Policy was introduced by Francisco De Almeida

2. Tobacco and cashew nuts were introduced to India by Nino da Cunha

Which of the above statements is/are correct?

a) 1 Only
b) 2 Only
c) Both 1 and 2
d) Neither 1 nor 2
Correct Answer: A
Your Answer: C
Explanation

Solution (a)

Basic Information:

Statement 1 Statement 2

Correct Incorrect

IASbaba
Score:
Web: http://ilp.iasbaba.com/
41.00 /
Email: ilp@iasbaba.com
Page 104 200
ILP FRESHERS- 2020 TEST
Exam Title :
41 ...
Email
Contact

Tobacco and cashew nuts were


The blue water Policy was introduced by
introduced to India by Alfenso de
Francisco De Almeida in the vision of
Albuquerque. He was the one who
making Portuguese as the master of
acquired Goa from sulthan of Bijapur in
Indian ocean
1510

QUESTION 97. MjI2MTgwK1BBUlRIK3BhcnRodW1yYWxpeWE1NUBnbWFpbC5jb20rODE0MDA3MTI2OFFVR


NUSU9OIDk2
Consider the following statements:

1. Abdulla Khan and Hussain Ali are known as king makers during Mughal period

2. Jahander shah abolished Izara

Which of the above statements is/are incorrect ?

a) 1 Only
b) 2 Only
c) Both 1 and 2
d) Neither 1 nor 2
Correct Answer: B
Your Answer: Unanswered
Explanation

Solution (b)

Basic Information:

After the death of Bahadur shah –I, the eldest son of Aurangzeb, Jahandar Shah became the
emperor with the help of Zulfikar khan

Statement 1 Statement 2

Correct Incorrect

Abdulla Khan and Hussain Ali are known


as king makers during Mughal period. Jahander shah abolished Jaziya. His
With the help of them Farrukhsiyar killed minister Zulfikar khan introduced Izara
jahander Shah and came into power

QUESTION 98. MjI2MTgwK1BBUlRIK3BhcnRodW1yYWxpeWE1NUBnbWFpbC5jb20rODE0MDA3MTI2OFFVR


NUSU9OIDk3
Consider the following statements:

IASbaba
Score:
Web: http://ilp.iasbaba.com/
41.00 /
Email: ilp@iasbaba.com
Page 105 200
ILP FRESHERS- 2020 TEST
Exam Title :
41 ...
Email
Contact

1. The Dutch founded their first factory in Masulipatanam in 1623 whereas the English founded
their factory at Masulipatanam in 1605

2. The English Company was issued Golden Farman by Sulthan of Golconda in 1632 226180

Which of the above statements is/are correct?

a) 1 Only
b) 2 Only
c) Both 1 and 2
d) Neither 1 nor 2
Correct Answer: B
Your Answer: Unanswered
Explanation

Solution (b)

Basic Information:

Portuguese came to India in 1498, after that English East India Company in 1600, Dutch East
India Company in 1602, Danish East India Company in 1616, French East India Company in
1664. Captain Hawkins arrived in the court of Jahangir but the mission to establish Surat didn’t
succeed due to opposition from Portuguese.

Statement 1 Statement 2

Incorrect Correct

The English Company was issued Golden


The Dutch founded their first factory in Farman by Sulthan of Golconda in 1632.
Masulipatanam in 1605 whereas the According to Golden Farman on the
English founded there factory at payment of 500 pagodas a year, they
Masulipatanam in 1611 earned the privilege of trading freely in
the ports of Golconda

QUESTION 99. MjI2MTgwK1BBUlRIK3BhcnRodW1yYWxpeWE1NUBnbWFpbC5jb20rODE0MDA3MTI2OFFVR


NUSU9OIDk4
Consider the following statements:

1. First Burma war ended with the treaty of Yandabo

2. Hyder ali died during the third Anglo-mysore war which made Tippu to carry the war

Which of the above statements is/are correct?

a) 1 Only

IASbaba
Score:
Web: http://ilp.iasbaba.com/
41.00 /
Email: ilp@iasbaba.com
Page 106 200
ILP FRESHERS- 2020 TEST
Exam Title :
41 ...
Email
Contact

b) 2 Only
c) Both 1 and 2
d) Neither 1 nor 2
Correct Answer: A
Your Answer: C
Explanation

Solution (a)

Basic Information:

Statement 1 Statement 2

Correct Incorrect

Hyder ali died of cancer during the


second Anglo-mysore war which made
First Burma war(1824-26) ended in 1826
Tippu to carry the war. Second Anglo-
with the treaty of Yandabo
Mysore war ended with the treaty of
Mangalore.

QUESTION 100. MjI2MTgwK1BBUlRIK3BhcnRodW1yYWxpeWE1NUBnbWFpbC5jb20rODE0MDA3MTI2OFFV


NUSU9OIDk5
Who was the Governor General who fought Second Anglo-Sikh war?

a) Lord Dalhousie
b) Lord Cornwallis
c) Lord Wellesly
d) Lord Canning
Correct Answer: A
Your Answer: Unanswered
Explanation

Solution (a)

Basic Information:

• Lord Dalhousie worked as a Governor General 1848-1856 during which he fought Second
Anglo-Sikh war which was during 1848-49.

• Lord Cornwallis (1786-1793) fought third Anglo-Mysore war.

• Lord wellesly (1798-1805) fought fourth Mysore war and Second Maratha war

• Lord Canning(1856-57) Revolt of 1857 and he acted as a Viceroy in between 1858-1862

IASbaba
Score:
Web: http://ilp.iasbaba.com/
41.00 /
Email: ilp@iasbaba.com
Page 107 200

You might also like